Menu Expand
SD - GI/Liver Secrets Plus E-Book

SD - GI/Liver Secrets Plus E-Book

Peter R McNally

(2010)

Abstract

GI/Liver Secrets Plus, 4th Edition, by Peter R. McNally, DO, FACP, FACG - a volume in the popular Secrets Series® - uses a convenient Q&A approach to provide rapid reference and review of today's most common GI and liver disorders and their management. An expanded size and layout, user-friendly two-color page layout, question-and-answer approach, and list of the "Top 100 GI/Liver Secrets" make it a perfect concise board review tool and a handy clinical reference. Updated coverage throughout equips you with all of the most current and essential knowledge in the field.

    • Uses bulleted lists, tables, boxes, short answers, and a highly detailed index to expedite reference.
    • Includes Key Points and memory aids in each chapter that make it perfect for board review or clinical reference.
    • Covers all of today's most common GI and liver disorders and their management.
    • Features the new "Secrets PLUS" format - with an expanded size and layout, more information, and more visual elements - for easier review and an overall enhanced reference experience.
    • Contains new images and illustrations to provide more detail and offer a clearer picture of what is seen in practice.
    • Incorporates revisions throughout to provide you with an up-to-date overview of gastroenterology and hepatology, including new chapters on Esophageal and Stomach Pathology, Pathology of the Lower GI Tract, Gastroesophageal Reflux and Esophageal Hernias, and Surgery of Achalasia and Esophageal Cancer.
    • Offers reorganized and expanded sections covering the bowels and colon for more specific and easier reference: Small and Large Bowel Disorders; Colon Disorders; and General Symptoms and Conditions.

Table of Contents

Section Title Page Action Price
Front cover Cover
Gi/Liversecrets Plus iii
Copyright page iv
Dedication v
Contributors vii
Preface xiii
Contents xv
Top 100 Secrets 1
Chapter 1: Swallowing Disorders and Dysphagia 7
1. What is the most difficult substance to swallow? 7
2. What sensory cues elicit swallowing? 7
3. What is the difference between globus sensation (globus hystericus) and dysphagia? 7
4. What are common etiologies of globus sensation? 7
5. Do patients accurately localize the site of dysphagia? 7
6. What are the differences between esophageal and oropharyngeal dysphagia? 7
7. What symptoms can be seen in oropharyngeal dysphagia? 7
8. What are the causes of oropharyngeal dysphagia? 8
9. What causes oropharyngeal dysphagia in the elderly? 8
10. Why is a brainstem stroke more likely to cause severe oropharyngeal dysphagia than a hemispheric stroke? 8
11. When is it appropriate to evaluate stroke-related dysphagia? 8
12. Is a barium swallow examination adequate to evaluate oropharyngeal dysphagia? 8
13. What is the characteristic feature of dysphagia in myasthenia gravis? 8
14. Why is simultaneous involvement of the oropharynx and esophagus extremely unusual for any disease process other than infection? 9
15. What is Zenker diverticulum? 9
16. Are Zenker diverticula the result of an obstructive or a propulsive defect? 9
17. What are the treatment options for Zenker diverticula? 10
18. How does flexible endoscopic therapy differ from standard surgical therapies? 10
19. What are the early complications following endoscopy therapy for Zenker diverticulum? 10
20. What are the indications and late risks of a cricopharyngeal myotomy? 10
21. When should you consider performing flexible endoscopic therapy for Zenker diverticula? 10
22. What is the differential diagnosis of dysphagia in a patient who has had surgery, radiation, and chemotherapy for head and neck cancer? 11
23. Are swallowing disorders related to an increased morbidity and mortality? 11
24. What therapies can be used to improve swallowing? 11
25. Which patients are ideal candidates for swallow therapy? 11
26. What are the etiologies of dysphagia in gastroesophageal reflux disease? 11
27. What are the common symptoms and causes of xerostomia? 11
28. Why is cricopharyngeal achalasia a misnomer? How does it differ from classic achalasia? 11
29. When is botulinum toxin (BTx) used for dysphagia? 11
Websites 12
Bibliography 12
Chapter 2: Gastroesophageal Reflux Disease 13
1. What is gastroesophageal reflux disease (GERD)? How common is it? 13
2. What are the typical symptoms of GERD? 13
3. Is gastrointestinal (GI) hemorrhage a common symptom of GERD? 13
4. What is odynophagia? Is it a common symptom of GERD? 13
5. What clues about GERD can be gleaned from the physical exam? 13
6. Do healthy persons have GERD? 14
7. How can swallowing and salivary production be associated with GERD? 14
8. What are the two defective anatomic mechanisms in patients with GERD? 14
9. What clearance defects are associated with GERD? 14
10. How may the GE barrier be compromised? 14
11. What foods and medications influence resting LES pressure? 14
12. What other medical conditions may mimic symptoms of GERD? 15
13. What medical condition clinically presents with dysphagia and is often mistaken for GERD? 15
14. How can GERD be distinguished from coronary artery disease? 15
15. How should patients with symptoms of GERD be evaluated? 15
16. Describe a commonly used endoscopic grading system for GERD 15
17. What are the more sophisticated esophageal function tests? How can they be used appropriately in the evaluation of patients with GERD? 15
18. Do all patients with GERD need esophageal function testing? 15
19. What is the use of multichannel intraluminal impedance and pH (MII-pH) technology in the evaluation of GERD? 15
20. When is ambulatory esophageal pH monitoring helpful? 16
21. When are esophageal manometry and scintiscanning helpful? 16
22. Define the various types of medical therapy for GERD and give a logical approach to prescription therapy for patients with longstanding GERD 16
23. Describe the commonly recommended approach to graded treatment of GERD 17
24. Do patients scheduled for surgical antireflux procedures need to undergo sophisticated esophageal function testing before surgery? 17
25. What are some of the new endoscopic treatments for GERD? 17
26. How should esophageal strictures be managed? 17
27. What is Barrett’s esophagus? How is it managed? 17
28. List some of the atypical symptoms and signs of GERD 17
29. Is there an association between obstructive sleep apnea (OSA) and GERD? 17
30. Does the presence of heartburn symptoms predict a GERD-related cough etiology? 17
31. What is the best method to evaluate for possible GERD-related cough? 18
32. What laryngeal conditions are associated with GERD? 18
33. How often do people with EPR and hoarseness relate symptoms of heartburn? 18
34. What is the most efficient, cost-effective method to evaluate hoarse patients for EPR? 18
35. Can GE reflux worsen asthma? 18
36. How does GE reflux worsen asthma? 18
37. What cytochrome P-450 (CYP-450) systems are involved in the metabolism of PPIs? 18
38. How do esomeprazole (Nexium) and omeprazole (Prilosec) differ? 18
39. Which patients with GERD should be considered for a surgical antireflux procedure? 19
40. Which patients are poor candidates for a surgical antireflux procedure? 19
Websites 19
Bibliography 19
Chapter 3: Esophageal Causes Of Chest Pain 20
1. When should the clinician consider an esophageal cause of chest pain? 20
2. Does history help to discriminate cardiac from esophageal chest pain? 20
3. Does a normal coronary angiogram exclude all cardiac diagnoses? 20
4. What are the noncardiac causes of chest pain? How common are they? 20
5. Because GERD is the most likely diagnosis, is a trial of acid suppression acceptable? 20
6. What is the most useful esophageal investigation? 21
7. How is esophageal pH monitoring performed? 21
8. What abnormalities may be found with pH monitoring? 21
9. If reflux has been excluded, which esophageal motility abnormalities may be found in patients with chest pain? 22
10. How can esophageal motility abnormalities cause chest pain? 23
11. Can esophageal pain be provoked during testing? 23
12. How does provocation testing compare with combined ambulatory monitoring of both motility and pH? 24
13. Are there any emerging technologies for investigation of UCP? 24
14. What is visceral hypersensitivity? Define the irritable esophagus 24
15. Does UCP have a psychological component? 24
16. What are the treatment options for nonreflux esophageal chest pain? 24
17. Are there any emerging treatment options? 25
18. Can abnormal belching or aerophagia cause chest pain? 25
19. What is the prognosis for patients with UCP? 25
Bibliography 25
Chapter 4: Achalasia 27
1. Define achalasia 27
2. How common is achalasia? 27
3. What is vigorous achalasia? 27
4. What is the relationship between diffuse esophageal spasm (DES) and achalasia? 27
5. What is the major pathologic lesion in achalasia, and how does it produce the disease? 27
6. What is the suspected cause of achalasia? 27
7. Is achalasia an acquired or a congenital disease? 28
8. Describe the dysphagia associated with achalasia 28
9. Are there any other symptoms associated with achalasia? 28
10. Does achalasia involve any other parts of the gastrointestinal tract? 28
11. What is the best way to diagnose achalasia? 28
12. What are the characteristic radiologic features of achalasia? 28
13. What is required for the manometric diagnosis of achalasia? 28
14. What is the most important potential pitfall in the manometric diagnosis of achalasia? 28
15. Describe the typical endoscopic features of achalasia 29
16. What is the difference between secondary achalasia and pseudo-achalasia? 29
17. How can pseudo-achalasia be diagnosed? 29
18. Is achalasia a premalignant condition? 29
19. Should patients undergo periodic endoscopic surveillance? 29
20. What are the various treatment options available for achalasia? Describe their rationale 29
21. Discuss the various pharmacologic options available for the palliation of achalasia 30
22. What does Viagra have to do with achalasia? 30
23. What is the single most permanent treatment of achalasia? 30
24. What is the major problem with surgery? 30
25. How can postoperative GERD be avoided? 30
26. How is balloon dilation of the LES accomplished? 30
27. What can be done if symptoms do not respond to the first dilation? 30
28. What are the results of pneumatic dilation? 30
29. How does pneumatic dilation compare with surgery? 31
30. What is the major disadvantage of forceful dilation? How can it be prevented? 31
31. How is perforation treated in patients with achalasia? 31
32. Which patients are particularly likely to respond to dilation? 31
33. What objective parameters should be followed after dilation? 31
34. How is botulinum toxin type A (BTxA) injection administered? 31
35. What are the results of BTxA treatment? 31
36. What are the major drawbacks of BTx treatment? 31
37. What is the overall best treatment for achalasia? 32
Acknowledgment 32
Websites 32
Bibliogrpahy 32
Chapter 5: Esophageal Cancer 33
1. What is the incidence of esophageal cancer in the United States and is it changing? 33
2. What are the risk factors for the development of esophageal cancer? 33
3. What are the current recommendations for screening and surveillance of esophageal cancer in patients at risk? 33
4. How is esophageal cancer diagnosed and staged? 33
5. Discuss the role of endoscopic ultrasound in the diagnosis and staging of esophageal cancer? 34
6. How is esophageal cancer staged? Why is staging important? 34
7. What is the prognosis of esophageal cancer? 35
8. What is the current management of esophageal cancer? 35
Treatment of Limited Disease (Stage I) 35
Treatment of Extensive Disease (Stage IIA–III) 35
Treatment of Distant Metastases (Stage IV) 35
9. What are the endoscopic methods for the palliation of esophageal cancer? 35
10. What does the future hold for patients at risk for the development ofesophageal cancer? 36
Websites 36
Bibliography 36
Chapter 6: The Esophagus: Anomalies, Infections, and Nonacid Injuries 37
1. A patient with iron deficiency anemia and dysphagia is found to have a web by barium studies. What disorder must be considered? 37
2. What is the best therapy for the dysphagia? 37
3. What is the best way to confirm a suspected web? 37
4. For which cancer are patients with esophageal webs reportedly at increased risk? 37
5. Describe the two types of esophageal rings 37
6. What causes esophageal A and B rings? 38
7. Are all Schatzki rings symptomatic? What is the typical history of the symptomatic patient? 38
8. How are Schatzki rings treated? 38
9. Describe the three types of esophageal diverticula 38
10. What is the typical history of Zenker diverticulum? 38
11. What causes Zenker diverticulum? 38
12. What is Killian dehiscence? 39
13. How is Zenker diverticulum treated? 39
14. Why are midesophageal diverticula called traction diverticula? 39
15. Should all epiphrenic diverticula be surgically treated? 39
16. Is surgery required for the cricopharyngeal bar or cricopharyngeal achalasia that radiologists sometimes describe? 40
17. Define dysphagia lusoria. What is the most common type? 40
18. What sort of preoperative evaluation should be done in patients with dysphagia lusoria? 40
19. What causes esophageal atresia with tracheoesophageal fistula? 40
20. What is the most common type of esophageal atresia with TE fistula? Describe its presentations 40
21. What is the least common type of esophageal atresia with TE fistula? Describe its presentation 40
22. How is esophageal atresia with TE fistula treated? 40
23. What is allergic esophagitis? 40
24. What is intramural pseudo-diverticulosis? 41
Esophageal Infections 41
25. What organisms are most commonly identified in esophageal infections? 41
26. What are the typical presenting symptoms in patients with infectious esophagitis? 41
27. What is the most common cause of infectious esophagitis in the generalpopulation? 41
28. List medical conditions known to predispose a person toCandida esophagitis. 42
29. What commonly used drugs are associated with fungalesophagitis? 42
30. How is Candida esophagitis treated? 42
31. Should empiric therapy for Candida esophagitis be considered in an at-risk patientpresenting with typical symptoms of esophageal infection? 42
32. What is the most common viral pathogen-causing esophagitis? 42
33. What is the most common cause of viral esophagitis in patients with normalimmunity? 42
34. How is viral esophagitis diagnosed? 42
35. Differentiate HSV and CMV esophagitis endoscopically. 42
36. How is HSV esophagitis treated? 43
37. Discuss the treatment of CMV esophagitis. 43
38. Does it make sense to stain for acid-fast organisms in evaluating esophagealulcers? 43
39. Can the diagnosis of Chagas disease be based on classic manometric findingsand confirmed by histologic evaluation of deep mucosal biopsies from the distalesophagus? 43
Pill And Corrosive Esophageal Injury 43
40. Who is affected by pill-induced esophageal injury? 43
41. What factors contribute to esophageal retention of pills? 43
42. What are the risk factors for pill-induced injury? 43
43. Describe the typical presentation of patients with pill-induced injury. 44
44. How is the diagnosis of pill-induced esophageal injurymade? 44
45. What does the typical pill-induced lesion look like at time of endoscopy? 44
46. How can you remember the many medications that can cause pill-inducedesophagitis? 44
47. Where are the areas of physiologic narrowing of the esophagus? 44
48. What are the options for treating pill-induced esophageal injury? 45
49. Discuss the epidemiology of caustic ingestion in the United States. 45
50. What are the common caustic agents? Where are they found? 45
51. Describe the pathophysiology of acute alkali esophagitis. 45
52. The severity of caustic injury to the esophagus can be graded as first, second, orthird degree, using the following system. 45
53. Describe how an endoscopic grading system guides in management and prognosis ofthe patient with corrosive injection. 45
54. What is the cancer risk to a patient with stricture after lye ingestion? 45
55. Describe the emergency department management of a patientwith caustic ingestion. 46
56. What is the role of endoscopic evaluation in patients withcaustic ingestion? 46
Websites 46
Bibliography 46
Chapter 7: Barrett’s Esophagus 48
1. What is Barrett’s esophagus? 48
2. How is Barrett’s esophagus diagnosed? 48
3. Why is Barrett’s esophagus important? 48
4. Does short-segment Barrett’s esophagus need to be identified? 48
5. What is the risk of cancer associated with Barrett’s esophagus? 48
6. Who should be screened for Barrett’s esophagus? 48
7. What is the therapy for Barrett’s esophagus? 48
Goals of Therapy for Barrett’s Esophagus 49
8. Does Barrett’s esophagus reverse with medical therapy? 49
9. Does Barrett’s esophagus reverse with surgical therapy? 49
10. What is the appropriate surveillance of patients with Barrett’s esophagus? 49
11. Summarize the evolution of Barrett’s esophagus to adenocarcinoma. 49
12. Describe the management of HGD. 49
13. Can the development of adenocarcinoma of the esophagus be prevented in patientswith Barrett’s esophagus? 49
14. What advantages can we anticipate in the management of Barrett’s esophagus? 50
Websites 50
Bibliography 50
Chapter 8: Esophageal and Stomach Pathology 51
Esophagus 51
1. Describe a normal esophagus lining. 51
2. What are the histologic features of gastroesophageal reflux disease (GERD) andeosinophilic esophagitis (EE)? 51
3. Discuss the infectious causes of esophagitis. 52
4. What is the most important differential to beconsidered in biopsy samples to evaluategraft-versus-host disease (GVHD)? 53
5. What is the histologic prevalence of esophageal Crohn’s disease in endoscopicallynormal studies? 53
6. What are other miscellaneous esophageal conditions? 53
7. List the dermatologic conditions that can affect the esophagus. 53
8. Discuss the histology of Barrett’s esophagus and the grading of dysplasia. 53
9. What histologic patterns can be seen in the biopsy samples from the GE junctionthat do not show typical endoscopic findings of Barrett’s esophagus? 54
10. What is the differential diagnosis of esophageal polypoid lesions? 54
11. What are the histologic features of the mucosal lining in different parts of the stomach? 56
12. What are the histologic patterns of gastritis? 56
13. What are the various histologic manifestations of Helicobacter pylori–associatedgastritis? 56
14. What is Helicobacter heilmannii–associatedgastritis? 57
15. What are the types of chronic atrophicgastritis, and how do these differ histologically? 57
16. What are the salient histologic features of chemical/reactive gastropathy? 57
17. What is lymphocytic gastritis, and with which disease processes is it associated? 57
18. What is the differential diagnosis ofgranulomatous gastritis? 58
19. What are the histologic featuressuggestive of gastric Crohn’s disease? 58
20. Histologically, how do you differentiategastric antral vascular ectasia (GAVE),portal hypertensive gastropathy,Dieulafoy lesion, and radiation injury? 58
21. What are the histologic features of giant mucosal folds seen in Ménétrier diseaseand Zöllinger-Ellison syndrome? 58
22. What are the histologic features of gastricpolyps/polypoid lesions? 59
23. Compare gastric dysplasia and adenoma. 59
24. What are the histologic types ofgastric adenocarcinoma? 60
25. What is the histologic classification of neuroendocrine neoplasms of the stomach? 60
26. What is the differential diagnosis of gastric stromal tumors? 61
27. What are the different types of gastric lymphoma? 61
Acknowledgments 61
Websites 61
Bibliography 61
Chapter 9: Gastritis 63
1. What is gastritis? 63
2. What are the symptoms of gastritis? 63
3. How is gastritis classified? 63
4. What is the endoscopic and histologic appearance of reactive and chronic gastritis? 63
5. How do we treat reactive gastritis, and what can be done to prevent it? 64
6. What are the characteristics of stress gastritis, and how should it be treated? 64
7. What is the most common etiology of chronic gastritis? 64
8. How is chronic gastritis secondary to H. pylori infection diagnosed? 64
9. How is chronic gastritis secondary to H. pylori infection treated? 65
10. What are the long-term implications of H. pylori infection? 65
11. What are the other types of chronic gastritis? 65
12. What is autoimmune gastritis? 65
13. What is lymphocytic gastritis? 65
14. What is chronic nonspecific gastritis? 65
15. What is Ménétrier disease, and how does it differ from the other special forms of chronic gastritis? 65
16. Describe bile reflux gastritis 66
17. In what circumstances do granulomatous and eosinophilic gastritis occur? 66
Bibliography 66
Chapter 10: Gastric Cancer 68
1. What are the histologic types of gastric cancer? 68
2. What is a signet ring cell carcinoma? 68
3. What is the ethnic and geographic distribution of distal gastric adenocarcinoma? 68
4. What is the role of diet in the development of gastric cancer? 68
5. What inherited genetic alterations are associated with gastric adenocarcinoma? 68
6. What is the role of H. pylori in gastric adenocarcinoma? 68
7. What mechanism is proposed for H. pylori causing an increased risk of gastric cancer? 69
8. What is the role of achlorhydria in gastric cancer? 69
9. Should H. pylori infection be eradicated to prevent gastric cancer from occurring? 69
10. Who should be screened for gastric cancer? 69
11. What is gastric stump cancer? 69
12. What is early gastric cancer? 69
13. How is the incidence of gastric adenocarcinoma changing? 69
14. What is the staging scheme for gastric adenocarcinoma? 70
15. How does staging help in treating gastric cancer? 70
16. What is the role of endoscopic ultrasonography in staging gastric cancer? 70
17. What is the role of endoscopy in the treatment of early gastric cancer? 70
18. What is the role of surgery in treating localized gastric adenocarcinoma? 70
19. What is the role of neoadjuvant therapy in gastric adenocarcinoma? 70
20. What is the role of adjuvant therapy in gastric adenocarcinoma? 71
21. What is the usual therapy for metastatic gastric adenocarcinoma? 71
22. What is a MALT lymphoma? 71
23. What is special about gastric MALT lymphomas? 71
24. What is the role of antibiotic therapy in gastric MALT lymphomas? 71
25. Describe the staging scheme for gastric lymphoma. 71
26. What is the best therapy for aggressive (non-MALT) gastric lymphoma? 71
27. What are gastric carcinoid tumors? 71
28. What causes gastric carcinoid tumors? 71
29. What should be done when a gastric carcinoid has been identified? 71
30. What is a gastric GIST? 72
Websites 72
Bibliography 72
Chapter 11: Helicobacter Pylori andPeptic Ulcer Disease 73
1. Why is Helicobacter pylori a unique bacterium? 73
2. What is the prevalence of H. pylori? 73
3. How is infection transmitted? 73
4. Where in the gastrointestinal (GI) tract does H. pylori live? 74
5. How does H. pylori produce mucosal damage? 74
6. What endoscopy-based (invasive) tests can be used to diagnose H. pylori infection? 74
7. How is H. pylori diagnosed noninvasively? 74
8. What is the association of H. pylori with histologic gastritis? 75
9. What is the association of H. pylori with peptic ulcer disease? 75
10. What may cause ulcers besides H. pylori? 75
11. Does H. pylori cause symptoms in patients with functional dyspepsia? 75
12. Does H. pylori play a role in gastric cancer? 76
13. In what situation is it appropriate to eradicate H. pylori infection? 76
14. What treatment regimens have been used to eradicate H. pylori? 76
15. Is reinfection a common problem? 77
16. What is the role of vaccination in the prevention of H. pylori? 77
17. What role does H. pylori play in gastroesophageal reflux disease (GERD)? 77
18. Is H. pylori associated with any diseases outside the GI tract in humans? 77
Websites 78
Bibliography 78
Chapter 12: Gastric Polyps and Thickened Gastric Folds 79
1. What are gastric polyps, and what are the most commonly observed types of gastric polyps? 79
2. Describe the endoscopic features typical of each type of gastric polyp. 79
3. Describe the histologic features of each type of gastric polyp. 79
4. What is the risk of malignancy associated with gastric polyps? 79
5. How should gastric polyps be managed? 80
6. Is surveillance indicated for patients with gastric polyps? 80
7. Describe the relationships between gastric polyps and other conditions. 80
8. What is meant by thickened gastric folds? 80
9. List the differential diagnosis for intrinsic causes of thickened gastric folds. 80
10. What systemic diseases may be associated with thickened gastric folds or granulomatous gastritis? 80
11. Endoscopic ultrasound (EUS) displays the gastric wall in five alternating hyperechoic and hypoechoic bands. Histologi ... 81
12. Describe the role of EUS in the evaluation of thickened gastric folds. 81
13. What are the clinical features of high-grade non-Hodgkin’s gastric lymphoma? 81
14. Define MALToma. 81
15. How are MALTomas managed? 82
16. Define Ménétrier disease. 82
17. How is Ménétrier disease different in children and adults? 83
18. What is the differential diagnosis for a subepithelial mass seen on endoscopy? 83
19. What role does EUS play in evaluating submucosal lesions? 83
20. What is a GIST? 84
21. A 65-year-old woman presents with self-limited, coffee-grounds emesis. Endoscopy reveals a single, pedunculated, 1-cm ... 84
22. A patient with FAP has multiple gastric polyps on surveillance endoscopy. What is the most likely histology of such p ... 84
23. Describe the manifestations of gastric polyps in the other hereditary GI polyposis syndromes. 84
24. A 40-year-old man has a history of chronic pancreatitis complicated by pseudocysts requiring drainage. He presents wi ... 84
25. A 65-year-old woman is referred for evaluation of chronic iron deficiency anemia and hemoccult-positive stool. Colono ... 84
Bibliography 85
Chapter 13: Gastroparesis 86
1. Define gastroparesis. 86
2. What are the factors that determine gastric motility and emptying? 86
3. Describe the electric pacesetter in the stomach. 86
4. What is the migrating motor complex? 87
5. Describe gastric motility and emptying. 87
6. What is idiopathic gastroparesis? 87
7. What is diabetic gastroparesis? 87
8. What is the pathogenesis of diabetic gastroparesis? 88
9. What surgical procedures are associated with postoperative gastroparesis? 88
10. What conditions cause selective gastric motor dysfunction leading to gastroparesis? 88
11. Which disorders with diffuse abnormalities of gastrointestinal motor activity cause gastroparesis? 88
12. Which drugs affect gastric emptying? 89
13. List the conditions that have an established association with delayed gastric emptying. 89
14. What associations are likely to be important in gastroparesis? 89
15. Which part of the history and physical exam is important in establishing a diagnosis of gastroparesis? 90
16. What modalities are available for diagnosing gastroparesis? 90
17. Outline an approach to the diagnosis of gastroparesis. 90
18. Once gastroparesis is diagnosed, how should it be treated? 91
19. What are the complications of gastroparesis? 91
20. What is the surgical management for medically refractory gastroparesis? 91
21. What is the role of botulinum toxin injection in refractory gastroparesis? 91
22. What is gastric electrical stimulation? 92
23. What is the role of gastric electrical stimulation in refractory gastroparesis? 92
Websites 92
Bibliography 92
Chapter 14: Evaluation of Abnormal Liver Tests 94
1. What are liver tests? 94
2. What are the true liver function tests? 94
3. What is the difference between cholestatic and hepatocellular injury? 94
4. What is the most specific test for hepatocellular damage? 94
5. How is cholestatic injury best diagnosed? 94
6. What are serum transaminases? 94
7. How is ALT assessed? 95
8. How are normal and abnormal levels of ALT determined? 95
9. What makes the AP level rise? 95
10. What does an elevated bilirubin mean? 96
11. How is bilirubin level determined? 96
12. What tests are used to evaluate hemochromatosis? 96
13. Describe the role of a1-antitrypsin. 96
14. What three tests are used to diagnose a1-antitrypsin deficiency? 96
15. What is Wilson’s disease? 97
16. How is Wilson’s disease diagnosed? 97
17. Summarize the tests for common metabolic disorders of the liver. 97
18. What are autoimmune markers? 97
19. How are the common antibody tests performed and interpreted? 97
20. When should screening or diagnostic tests be ordered for patients with suspected liver disease? 98
21. What are noninvasive markers of fibrosis, and what is their utility? 98
22. What is the role of liver biopsy? 98
Websites 98
Bibliography 98
Chapter 15: Viral Hepatitis 100
1. What are the types of hepatitis viruses? 100
2. What is the difference between acute and chronic hepatitis? 100
3. How common is chronicity in hepatitis B? 100
4. When does chronic hepatitis D develop? 100
5. How common is chronic hepatitis C? 100
6. How are hepatitis viruses transmitted? 100
7. Describe the symptoms of hepatitis. 101
8. What biochemical abnormalities are associated with viral hepatitis? 101
9. What biochemical findings indicate chronic infection? 101
10. How is hepatitis A diagnosed? 101
11. How is hepatitis B diagnosed? 101
12. Describe the HBsAg and HBsAb tests. 102
13. How is the HBcAb test interpreted? 102
14. What do the HBeAg and HBeAb tests indicate? 102
15. Describe the bDNA assay. 102
16. What are HBV PCR assays? 102
17. How is hepatitis C diagnosed? 102
18. How is a positive result on the ELISA confirmed? 103
19. What nucleic acid assays are available for hepatitis C? 103
20. How is hepatitis D diagnosed? 103
21. How is hepatitis E diagnosed? 103
22. Are there other hepatitis viruses not yet discovered? 103
23. What is the treatment of acute hepatitis B and hepatitis D? 103
24. What is the treatment of acute hepatitis C? 103
25. Is chronic viral hepatitis treatable? 104
26. Does lamivudine have any clinical relevance in this day and age? 104
27. Which patients with chronic hepatitis B are candidates for therapy? 104
28. Describe the standard treatment and its side effects. 104
29. What is the response rate to treatment of chronic hepatitis B? 104
30. Describe the treatment of chronic hepatitis C. How effective is it? 104
31. Describe the side effects of ribavirin. 104
32. What is PEG-interferon? 105
33. What happens if the patient fails to clear the hepatitis C virus? 105
34. What alternative therapies for hepatitis C are under investigation? 105
35. How is hepatitis D treated? 105
36. Can hepatitis C be prevented? 105
Controversy 105
37. Should all patients with hepatitis C undergo liver biopsy? 105
Website 105
Bibliography 106
Chapter 16: Antiviral Therapy For Hepatitis C Infection 107
1. What are the indications for antiviral therapy in patients with chronic hepatitis C? 107
2. What is the recommended evaluation of patients with chronic hepatitis C before therapy is begun? 107
3. Should hepatitis C genotype testing be performed before initiation of therapy? 108
4. Is a liver biopsy mandatory before initiation of antiviral therapy? 108
5. What are the treatment options for hepatitis C infection? 108
6. How are the antiviral agents dosed? 108
7. How is response to antiviral therapy assessed? 108
8. How often should viral load be measured during treatment? 109
9. What pretreatment characteristics predict a favorable response to antiviral therapy? 109
10. What is the efficacy of combination peginterferon and ribavirin? 109
11. How can response to antiviral therapy be maximized? 109
12. What are the side effects of IFN therapy? How should the patient be monitored? 109
13. What are the side effects of ribavirin therapy? How should the patient be monitored? 110
14. What are the contraindications to IFN therapy? 110
15. What are the contraindications to ribavirin therapy? 110
16. Should patients with cirrhosis secondary to hepatitis C infection be treated with antiviral therapy? 110
17. Should patients with hepatitis C and normal liver enzyme levels be treated with antiviral therapy? 111
18. Should patients with HCV/HIV coinfection receive antiviral therapy for hepatitis C infection? 111
19. How should patients with HCV/HBV coinfection be treated? 111
20. What are the options for patients who did not respond to combination therapy with nonpegylated IFN and ribavirin? 111
21. What is the role of antiviral therapy in acute hepatitis C? 111
22. Are there new treatments on the horizon for hepatitis C? 112
Websites 112
Bibliography 112
Chapter 17: Antiviral Therapy For Hepatitis B 113
1. Is antiviral therapy recommended for acute hepatitis B? 113
2. Do all patients with chronic hepatitis B benefit from therapy? 113
3. How should the HBV-DNA PCR assay results be used to make therapy decisions? 113
4. Is liver biopsy required before therapy is started? 114
5. What are the options for treating chronic hepatitis B infection? 114
6. What are the endpoints of antiviral therapy? 114
7. What is the expected response to interferon therapy? 115
8. What is the expected response to oral nucleoside/nucleotide therapy? 115
9. What are the advantages of interferon therapy for chronic hepatitis B infection? 115
10. What are the disadvantages of interferon therapy? 115
11. Which parameters predict a good response to interferon therapy? 115
12. What are the advantages of oral nucleoside/nucleotide therapy? 116
13. What are the disadvantages of oral nucleoside/nucleotide therapy? 116
14. Should patients with advanced, decompensated cirrhosis secondary to hepatitis B receive antiviral therapy or be referred for liver transplantation without a trial of therapy? 116
15. How should response to therapy be monitored? 116
16. How is resistance diagnosed and how should it be managed? 116
17. Should patients with chronic hepatitis B be treated if they are to undergo immune suppression? 117
18. How should HBV infection be treated in patients coinfected with the human immunodeficiency virus (HIV)? 117
19. Should hepatitis B be treated during pregnancy? 117
Websites 118
Bibliography 118
Chapter 18: Autoimmune Hepatitis: Diagnosis And Pathogenesis 119
1. What is autoimmune hepatitis? 119
2. What are its predominant features? 119
3. What are the symptoms of autoimmune hepatitis? 120
4. What are the characteristic histologic findings in autoimmune hepatitis? 120
5. Can autoimmune hepatitis have a fulminant presentation? 121
6. Are there patients who may be underdiagnosed? 121
7. What are the different types of autoimmune hepatitis? 122
8. What are the clinical criteria for diagnosis? 122
9. What are the diagnostic scoring systems and how are they used? 123
10. What is the standard serologic battery for diagnosis? 124
11. What serologic assays are best for detecting the standard autoantibodies? 125
12. What other autoantibodies may have diagnostic and prognostic importance? 125
13. What investigational antibodies have promise as clinical tools? 126
14. What is the significance of antimitochondrial antibodies in autoimmune hepatitis? 126
15. Can autoimmune hepatitis exist in the absence of conventional autoantibodies? 126
16. What is the appropriate testing sequence for the autoantibody determinations? 127
17. In what other clinical situation should autoimmune hepatitis be considered? 127
18. What are the variant (overlap) syndromes of autoimmune hepatitis? 127
19. Is autoimmune hepatitis in children different from that of adults? 128
20. What are the pathogenic mechanisms? 128
21. What are the autoantigens? 129
22. Can viruses cause autoimmune hepatitis? 129
23. Can drugs cause autoimmune hepatitis? 129
24. Are there genetic predispositions for autoimmune hepatitis? 129
25. Does autoimmune hepatitis have a Mendelian pattern of inheritance? 129
26. What are the susceptibility alleles? 129
27. How do different susceptibility alleles produce the same disease? 130
28. How do regional factors affect disease occurrence? 130
29. Why do patients with the same HLA have different clinical phenotypes? 130
30. Do the HLA phenotypes influence disease expression and outcome? 130
31. Should HLA typing be part of the standard diagnostic algorithm? 130
Website 131
Bibliography 131
Chapter 19: Autoimmune Hepatitis: Treatment 132
1. What therapies are effective for patients with autoimmune hepatitis? 132
2. How do the medications work? 133
3. What are the side effects of the medication? 133
4. Can azathioprine be used during pregnancy? 134
5. What are the indications for treatment? 134
6. What are the indices that reflect disease severity? 134
7. Are there any predictors of response to treatment? 135
8. Does the rapidity of the response to treatment have prognostic value? 135
9. What are the results of therapy? 135
10. What are the endpoints of treatment? 136
11. When should a liver biopsy be performed? 136
12. Does corticosteroid treatment prevent or reverse fibrosis? 136
13. What precautions can be undertaken to reduce the frequency side effects? 136
14. What is the most common treatment problem? 136
15. What are the consequences of relapse and retreatment? 136
16. Does determination of serum thiopurine methyltransferase activity predict azathioprine toxicity? 136
17. How should relapse be managed? 137
18. Do patients receiving azathioprine maintenance therapy ever get off treatment? 137
19. How should treatment failure be managed? 137
20. How effective is liver transplantation for decompensated disease? 138
21. What strategy is best for patients with drug toxicity or incomplete response? 139
22. Does hepatocellular carcinoma occur? 139
23. How are variant syndromes managed? 139
24. How are patients with mixed autoimmune and viral features managed? 139
25. What new drug therapies are promising? 140
26. What new site-specific molecular interventions are promising? 140
Website 141
Bibliography 141
Chapter 20: Primary Biliary Cirrhosis and Primary Sclerosing Cholangitis 142
1. Define primary biliary cirrhosis (PBC) and primary sclerosing cholangitis (PSC) 142
2. Is PBC an autoimmune disorder? 142
3. Is PSC an autoimmune disorder? 142
4. Do viral infections have a role in the development of PSC? 142
5. What are the clinical features of PBC and PSC? 142
6. What are the common findings on physical examination? 143
7. What diseases are associated with PBC? 143
8. What diseases are associated with PSC? 143
9. What important biochemical abnormalities are associated with PBC and PSC? 143
10. What is the lipid profile in patients with PBC? Are they at increased risk for developing coronary artery disease? 143
11. What serum autoantibodies are associated with PBC? 143
12. What serum autoantibodies are associated with PSC? 143
13. What are the cholangiographic features of the biliary tree in PSC? 144
14. Is it important to evaluate the biliary tree in PBC? 144
15. What are the hepatic histologic features of PBC and PSC? 144
16. Do asymptomatic patients with PBC have a normal life expectancy? 145
17. Do asymptomatic patients with PSC have a normal life expectancy? 145
18. What is the role of mathematical models in estimating survival for PBC and PSC? 145
19. What vitamin deficiencies are associated with PBC and PSC? 145
20. What bone disease is associated with PBC and PSC? 145
21. What liver-related complications are specific to PSC? 146
22. What is the differential diagnosis of PBC and PSC? 146
23. Define autoimmune cholangitis. How is it related to PBC? 146
24. What is meant by an overlap or a variant syndrome in PBC and PSC? 146
25. What is meant by small duct PSC? 146
26. Describe the treatment of pruritus in patients with PBC and PSC. 147
27. How is osteopenia treated in patients with PBC and PSC? 147
28. Describe the treatment of fat-soluble vitamin deficiency in PBC and PSC. 147
29. Do lipid-lowering agents have a role in treatment of PBC and PSC? 147
30. Describe the treatment of bacterial cholangitis in PSC. 147
31. What are the therapeutic options for biliary strictures in PSC? 147
32. Describe the role of transjugular intrahepatic portosystemic shunt (TIPS) in PBC and PSC. 148
33. What medical agents have been tried for the treatment of PBC? 148
34. What medical agents have been tried for the treatment of PSC? 148
35. Describe the mechanism of action for UDCA in PBC 148
36. What is the role of reconstructive biliary tract surgery in PSC? 148
37. Does proctocolectomy in patients with PSC and CUC favorably affect hepatobiliary disease? 148
38. Do patients with PBC and PSC have an increased risk for hepatocellular carcinoma (HCC)? 148
39. What is the role of liver transplantation in PBC and PSC? 149
40. Do PBC and PSC recur after liver transplantation? 149
41. What are the complications in PSC patients after liver transplantation? 149
Websites 149
Bibliography 149
Chapter 21: Hepatitis Vaccines and Immunoprophylaxis 151
1. Discuss the concept of immunization (vaccination) 151
2. Outline briefly the history of vaccination 151
3. Distinguish between active and passive immunization 151
4. What are the major categories of vaccines? 151
5. Describe the basic characteristics of immunoprophylaxis 151
6. Which Igs are available for human use? 151
7. Which viral agents are mainly responsible for acute and chronic viral hepatitis? 152
8. When should Ig be used? 152
9. What vaccines are available for hepatitis A? 152
10. Compare the major characteristics of VAQTA, HAVRIX, and TWINRIX 153
11. Who should be immunized against hepatitis A? 153
12. What side effects have been observed with the hepatitis A vaccine? 153
13. Do nonresponders to hepatitis A vaccine exist? 154
14. What is the lowest protective anti-HAV serum level after immunization? 154
15. Does the concurrent administration of hepatitis A vaccine influence the immune response to other traveler’s vaccines? 154
16. What kind of immunoprophylaxis is available for hepatitis B? 154
17. What is the recommended dose of HBIG for adults and children? 154
18. How many hepatitis B vaccines are available in the United States? Are they comparable? 154
19. What is the immunization schedule for HBV vaccine in adults and children? 154
20. What is the recommended regimen for infants born to HBsAg-positive mothers? 155
21. Is a booster needed after immunization? If so, how often? 155
22. Summarize the evidence for long-term immunization after vaccination. 155
23. Is it possible that the vaccine will not protect against HBV infection? 155
24. Is it harmful to give hepatitis B vaccine to known hepatitis B carriers? 156
25. Is it appropriate to use the therapeutic vaccines to Hepatitis B? 156
26. Is it possible to immunize people simultaneously against hepatitis A and B? 156
27. Is immunoprophylaxis advisable for hepatitis C? 156
28. Is a vaccine available for hepatitis E? 156
29. Should patients with chronic liver disease be immunized against hepatitis A and hepatitis B? 156
Website 156
Bibliography 156
Chapter 22: Pregnancy and Liver Disease 158
Normal Anatomical And Physiologic Changes During Pregnancy 158
1. What are the structural and functional hepatic adaptations during pregnancy? 158
2. Does liver function change during pregnancy? 158
Diseases During Pregnancy 158
3. Can gestational age differentiate between different liver diseases in pregnancy? 158
Coincident Occurrence 158
4. Can we assume the presence of chronic liver disease in a pregnant patient withangiomas and palmar erythema on physical examination and small esophagealvarices detected endoscopically? 158
5. What is the most common cause of jaundice in pregnancy? 158
6. How severe is the course of viral hepatitis acquired during pregnancy? 159
7. What signs and symptoms suggest the diagnosis of Budd-Chiari syndrome? 159
8. Is the serum ceruloplasmin level a good diagnostic marker in pregnant women atterm who are suspected of having Wilson disease? 159
9. Can we maintain a woman with Wilson disease on therapy duringpregnancy? 159
Intrahepatic Cholestsis Of Pregnancy 159
10. What is the most common liver disorder unique to pregnancy? 159
11. What is the major clinical manifestation of IHCP? 159
12. What biochemical changes are noted in IHCP? 159
13. What is the expected clinical and biochemical course after delivery for patientswith IHCP? 159
14. What is a possible cause for abnormal bleeding in a postpartum woman previouslydiagnosed with IHCP? What is the treatment? 160
15. What is the effect of IHCP on the fetus? 160
16. What is the therapy for IHCP? 160
17. Can IHCP recur? 160
18. What atypical signs and symptoms make the diagnosis of IHCP doubtful? 160
19. What biochemical changes suggest an alternate diagnosis? 160
Acute Fatty Liver Of Pregnancy 160
20. What are the clinical and laboratory features of AFLP? 160
21. How do we diagnose and treat AFLP? 160
22. Is biopsy pathognomonic for AFLP? 160
23. Describe the pathogenesis of AFLP. 161
24. What is the outcome of a child whose mother has AFLP? 161
25. Does AFLP recur in subsequent pregnancies? 161
26. Is genetic testing indicated in women diagnosed with AFLP? 161
Hemolysis, Elevated Liver Enzymes, And Low Platelets (Hellp) 161
27. What is the spectrum of liver involvement in preeclampsia? 161
28. How common is HELLP syndrome? 161
29. Describe the incidence and prognosis of spontaneous intrahepatic hemorrhage. 161
30. What findings typically lead to the diagnosis of HELLP syndrome? 161
31. What is the treatment for severe preeclamptic liver disease? 162
32. Does HELLP recur in subsequent pregnancies? 162
33. What information helps to differentiate AFLP from HELLP? 162
34. Is prospective screening necessary in pregnancies complicated by AFLP or HELLP? 162
Care Of Patients With Preexisting Liver Disease 162
35. What methods of contraception are available for patients with liver disease? 162
Management Of Underlying Liver Disease 163
36. How should patients with preexisting liver disease be managed if pregnancyoccurs? 163
Management Of Portal Hypertension 163
37. What are the effects of pregnancy on the mother with portal hypertension? 163
38. What is the effect of maternal portal hypertension on pregnancy? 163
Management In The Setting Of Orthotopic Liver Transplantation 163
39. When can a liver transplant recipient actively seek conception? 163
40. Is pregnancy possible after liver transplantation? 163
41. What are the possible complications of pregnancies occurring after livertransplantation? 163
42. What is recomme 164
43. What are pregnancy safety data regarding maintenance immunosuppressive agentsused in orthotopic liver transplantation (OLT)? 164
44. Is breastfeeding permitted after delivery in a liver transplant recipient? 164
45. Are immunosuppressive agents safe during pregnancy? 164
Prevention Of Vertical Transmission 164
46. How may vertical transmission of viral hepatitis A be prevented? 164
47. How may vertical transmission of viral hepatitis B be prevented? 164
48. What about vertical transmission of viral hepatitis C? 165
49. Is it possible to prevent vertical transmission of viral hepatitis D and G? 165
50. Are HCV-infected women allowed to breastfeed? 165
51. Does the mode of delivery influence hepatitis C transmission? 165
52. How can perinatal HCV infection be diagnosed? 165
Websites 165
Bibliography 165
Chapter 23: Rheumatologic Manifestations of Hepatobiliary Diseases 167
Viral Hepatitis 167
1. How often is viral hepatitis associated with rheumatic manifestations? 167
2. What are the most common extrahepatic rheumatologic manifestations of hepatitis Binfection? 167
3. Describe the clinical characteristics of the polyarthritis-dermatitis syndromeassociated with hepatitis B infection. 167
4. What is the typical presentation of hepatitis B–associated polyarteritis nodosa (PAN)? 167
5. How is PAN associated with hepatitis B antigenemia diagnosed? 167
6. What is the treatment of hepatitis B–associated PAN? 167
7. What are the most common hepatitis C virus (HCV)-related autoimmune disorders? 167
8. What is the relationship between viral hepatitis and cryoglobulinemia? 168
9. Describe the typical clinical features of cryoglobulinemia associated with hepatitisC infection. 168
Autoimmune And Other Liver Diseases 168
10. What is lupoid hepatitis? 168
11. To what degree is type I AIH similar to SLE? 169
12. What is the difference between anti-Sm and anti-SM antibodies? 169
13. List the common autoimmune diseases associated with primary biliarycirrhosis (PBC). 169
14. Compare and contrast the arthritis that may occur with PBC and rheumatoidarthritis. 169
15. What other musculoskeletal manifestations may occur in patients with PBC? 170
16. What autoantibodies commonly occur in patients with PBC? 170
17. How commonly does arthritis occur in patients with hereditary hemochromatosis? 170
18. Describe the radiographic features suggestive of hemochromatotic arthropathy. 170
19. What is the relationship between calcium pyrophosphate disease and hemochromatosis? 170
20. Discuss the genetics of hereditary hemochromatosis. 170
21. Compare and contrast the features of hemochromatotic arthropathy (HA) andrheumatoid arthritis (RA). 171
22. How effective is phlebotomy in halting the progression of hemochromatoticarthropathy? 171
23. What is the correlation between the severity of arthropathy and severity of liverdisease in hemochromatosis? 171
24. Why does hemochromatosis cause a degenerative arthritis? 171
25. What other musculoskeletal problems may occur in patients with hemochromatosis? 171
Bibliography 171
Chapter 24: Evaluation of Focal Liver Masses 173
1. Describe the initial workup for a patient with a liver mass 173
2. What tumor markers are useful in the evaluation of focal liver lesions? 173
3. What imaging modalities are used in the detection and characterization of focal liver masses? 174
4. What is the most common benign cause of a focal liver lesion? 174
5. Why is oral contraceptive use important in the differential diagnosis of focal liver masses? 174
6. Why is surgical resection of hepatic adenomas recommended? 174
7. What is focal nodular hyperplasia (FNH)? 174
8. List the differences between hepatic adenomas and FNH 175
9. What is the most frequent malignancy in the liver? 175
10. What is the most common primary liver cancer? 175
11. Describe the various presenting forms of HCC 175
12. What types of cirrhosis are most commonly associated with HCC? 175
13. What clinical and laboratory findings should raise suspicion for HCC? 175
14. What primary liver tumor occurs in young adults without underlying cirrhosis? 176
15. What factors predispose to the development of cholangiocarcinoma? 176
16. What is a Klatskin tumor? 176
17. When should liver transplantation be considered in patients with HCC? 176
18. When should resection be considered in patients with HCC? 176
19. What palliative therapies are available for the management of HCC? 176
20. Who should be screened for HCC? Describe a typical screening strategy 177
21. What benign tissue abnormality may simulate a focal liver mass? 177
22. What new imaging techniques are under development to evaluate focal liver masses? 177
23. Why is fine-needle biopsy of hepatic masses controversial? 177
24. What should be done when small incidental liver lesions are found? 177
25. Outline a logical approach to the evaluation of a focal hepatic mass. 178
Incidental Lesions 178
Symptomatic Lesions 178
Cirrhosis Or Risk Factors For Cholangiocarcinoma 178
History Of Malignancy 178
Website 178
Bibliography 178
Chapter 25: Drug-Induced Liver Disease 180
1. How common is drug-induced liver disease? 180
2. How are the three patterns of drug-induced liver injury distinguished? 180
3. Describe the typical chronologic association between drug exposure and onset of hepatitis or cholestasis. 180
4. What is the differential diagnosis of drug-induced liver disease? 180
5. Explain the two most common mechanisms of drug-induced liver injury 180
6. What variables appear to influence susceptibility to drug-induced hepatic injury? 180
7. Name the two most common causes of drug-induced liver disease 181
8. How is acetaminophen toxic to the liver? 181
9. At what dose is acetaminophen toxic? 181
10. How is acetaminophen toxicity treated? 181
11. Describe the clinical features of allergic hepatitis. 181
12. What drugs have been reported to cause chronic hepatitis and cirrhosis? 181
13. Name the two types of cholestatic drug-induced hepatic injury 181
14. List the common causes of drug-induced cholestasis 181
15. List the drugs associated with the mixed cholestatic-hepatitis type of liver injury 181
16. Which drugs cause the three types of drug-induced steatosis (fatty liver)? 181
17. Which three vascular injuries to the liver can be caused by drugs? 182
18. What are the three most common drug-induced hepatic neoplasms? 182
19. More than 50 drugs have been cited as causing hepatic granulomas. Name the most common 182
20. What antiarthritic drugs have been reported to cause liver injury? 182
21. How should patients receiving chronic methotrexate (MTX) be monitored for chronic hepatitis and cirrhosis? 182
22. What are the histologic grades of MTX liver injury? 183
23. Outline the recommendations for change in MTX therapy based on liver biopsy findings 183
24. What are the clinical findings of chlorzoxazone hepatotoxicity? 183
25. Which drugs commonly used to treat endocrine disease have been reported to cause liver injury? 183
26. What commonly used cardiovascular drugs have been reported to cause liver injury? 183
27. What are the clinical features of methyldopa (Aldomet) hepatocellular injury? 184
28. What commonly used antimicrobial agents have been shown to cause liver injury? 184
29. Who is at risk for liver toxicity from isoniazid (INH) therapy? 184
30. How is INH toxicity prevented? 184
31. What commonly used recreational drugs are associated with hepatotoxicity? 184
32. What anesthetic agents are associated with hepatocellular injury? 185
33. Can herbal therapies injure the liver? 185
Websites 185
Bibliography 185
Chapter 26: Alcoholic Liver Disease 186
1. How does the liver metabolize ethanol? 186
2. How common is alcohol abuse in the United States? 186
3. How can I screen a patient for alcoholism during an office visit? 186
4. What are the signs and symptoms of alcohol withdrawal syndrome? 187
5. How should I manage alcohol withdrawal in the alcoholic patient? 187
6. How is alcohol detoxification best managed? 187
7. What are the usual steps taken for in-hospital treatment of withdrawal? 187
8. What treatments help in the long term for alcohol dependency? 188
9. What are the different types of alcoholic liver disease (ALD)? 188
10. How does ALD differ from nonalcoholic fatty liver disease? 188
11. What is the natural history of ALD? 188
12. What is the epidemiology of ALD? 189
13. What is the pathogenesis of ALD? 189
14. What are the clinical findings in the patient with ALD? 190
15. What are the laboratory findings in patients with ALD? 190
16. Patients with alcoholic fatty liver may lack any laboratory signs, although aminotransferase levels and GGT can be elevated 190
17. How does radiographic imaging help in evaluation of the patient with ALD? 190
18. What are the characteristic histologic features of ALD? 190
19. What is the treatment for ALD? 191
20. What is the prognosis of patients with ALD? 191
21. Does HCV infection increase the risk of cirrhosis in the alcoholic? 192
22. Does alcoholic cirrhosis predispose a patient to development of HCC? 192
23. How should I screen patients with alcoholic cirrhosis for HCC? 192
24. Can the patient with end-stage ALD undergo liver transplantation? 192
Websites 193
Bibliography 193
Chapter 27: Vascular Liver Disease 194
1. Describe the principal vascular anatomy of the liver. 194
2. Describe the microcirculation of the liver. 194
3. What makes the liver resistant to ischemic and vascular disease? 195
4. What is Budd-Chiari syndrome? 195
5. How is BCS diagnosed? 196
6. Why do some patients with BCS have an enlarged caudate lobe? 196
7. What are the histopathologic findings in BCS? 196
8. What is the treatment for BCS? 196
9. Which patients with BCS should be considered for liver transplantation? 197
10. What is the pathogenesis of hepatic veno-occlusive disease (VOD)? 197
11. What are the clinical features of VOD? 197
12. How is VOD diagnosed? 197
13. Describe the clinical features of ischemic hepatitis 197
14. What is the pathogenesis of ischemic hepatitis? 197
15. What are the clinical manifestations of congestive hepatopathy? 197
16. What liver chemistry abnormalities are found in congestive hepatopathy? 198
17. Describe the pathologic changes associated with congestive hepatopathy 198
18. What is the most frequent vascular complication following liver transplantation? 198
19. What are the risk factors for portal vein thrombosis (PVT)? 198
20. What treatments options are available for PVT? 198
21. What is the most common vascular tumor of the liver? 198
22. What is hepatic hemangioendothelioma? 198
Bibliography 199
Chapter 28: Nonalcoholic Fatty Liver Disease and Nonalcoholic steatohepatitis 201
1. What is the difference between nonalcoholic fatty liver disease (NAFLD) and nonalcoholic steatohepatitis (NASH)? 201
2. How do patients with NAFLD present? 201
3. Describe the typical NAFLD patient 201
4. What is the prevalence of NAFLD and NASH? 202
5. How can you distinguish between NAFLD and NASH? 202
6. How is the severity of disease determined in patients with NASH? 203
7. Are there other causes of fatty liver beside insulin resistance/obesity/metabolic syndrome? 203
8. What is the relationship between hepatic steatosis and hepatitis C virus infection? 203
9. What is the cause (pathogenesis) of NAFLD, in particular NASH? 203
10. How do you treat patients with isolated fatty liver (i.e., NAFLD patients without histologic evidence of NASH)? 204
11. What is the optimal treatment of patients with biopsy-proved NASH? 204
12. How many patients diagnosed with NASH go on to require liver transplants? 205
13. What is the role of hepatic steatosis in liver transplant donors? 205
14. Does NAFLD/NASH recur after liver transplant? 205
Websites 205
Bibliography 206
Chapter 29: Liver Transplantation 207
1. What is the current basis for prioritizing patients for cadaveric transplantation? 207
2. Why was the MELD score developed? 207
3. For patients with chronic liver disease, when is the appropriate time to refer for liver transplantation? 207
4. Which patients with HCC are considered and prioritized for transplantation? 207
5. What can be done to improve the availability of donor organs? 208
6. Given the high waiting list mortality, is living donor liver transplantation (LDLT) an option? 208
7. What are the advantages and disadvantages of LDLT? 208
8. Who are potential recipients for LDLT? 208
9. List the diseases for which liver transplantation is performed 208
10. A 33-year-old man was diagnosed with acute hepatitis A 3 weeks ago. His jaundice has progressively worsened since then. Today, his wife found him to be mildly confused and brought him to the emergency department. What is the definition of acute liver fa 209
11. A 21-year-old woman is admitted following an overdose of acetaminophen. How do you determine whether she should be referred for liver transplantation? 209
12. Is human immunodeficiency virus (HIV) infection a contraindication to liver transplantation? 209
13. What conditions are considered contraindications to liver transplantation? 209
14. A 45-year-old man with end-stage liver disease is evaluated for liver transplantation. Which features of the patient’s psychosocial profile connote a good prognosis for continued abstinence from alcohol? 210
15. Which factors measured in the recipient prior to transplantation correlate with reduced postoperative survival? 210
16. Which immunosuppressants are used in liver transplantation? What are their mechanisms of action and side effects? 210
17. What is the typical immunosuppressive regimen? 211
18. A liver transplant patient has just sustained a grand mal seizure 36 hours post transplantation. The cyclosporine level is within acceptable limits. The patient is in a postictal state but has no obvious focal neurologic deficits. Which factors contribu 211
19. A patient, 3 weeks post transplantation, receives erythromycin for atypical pneumonia. Does this drug affect immunosuppressive therapy? 211
20. A patient who had an uncomplicated transplantation is noted to have rising liver enzyme levels on day 10 after transplantation. What is the differential diagnosis, and which tests should be obtained? 211
21. A patient with cirrhosis from chronic hepatitis C undergoes liver transplantation. Ten days later his liver enzymes increase. What are the histologic findings of acute rejection versus post-transplantation hepatitis C on liver biopsy? 212
22. Describe the other post-transplantation complications manifested by elevated liver enzymes 212
23. A patient having early allograft rejection is treated with a 7-day course of OKT3 returns 1 week later with headache, mild fatigue, low-grade fever, and increased liver enzymes. Is this OKT3 toxicity? 213
24. If the patient does not have OKT3 toxicity, what is the most likely diagnosis? 213
25. What are the clinical, biochemical, and histologic features of chronic rejection? 213
26. How often is it necessary to perform a second liver transplantation, and for what reasons are retransplantations performed? 213
27. Describe the long-term metabolic complications that occur in the liver transplant recipient 213
28. Are liver transplant recipients at increased risk to develop cancer? 214
29. A liver transplant patient comes to the emergency department complaining of cough and shortness of breath. How does one suspect, diagnose, and treat Pneumocystis jiroveci (carinii) pneumonia? 214
30. What factors contribute to metabolic bone disease after transplantation? 214
31. A patient who underwent liver transplantation for cirrhosis due to hepatitis C returns with persistently elevated liver enzymes. Liver biopsy reveals chronic active hepatitis but no cirrhosis. Should he be treated with interferon a and/or ribavirin? 214
32. Is retransplantation for recurrent hepatitis C recommended? 215
Bibliography 215
Chapter 30: Ascites 217
1. What are the most common causes of ascites? 217
2. Should a diagnostic tap be performed routinely on all patients with ascites at the time of admission to the hospital? 217
3. How should a diagnostic paracentesis be performed? 217
4. What tests should be routinely ordered on ascitic fluid? 218
5. Should a diagnostic thoracocentesis be performed in patients with cirrhosis and pleural hydrothorax? 218
6. Why is it useful to measure serum-ascites albumin gradient? 218
7. What are the causes of high (i.e., ≥1.1 g/dL) serum-ascites albumin gradients? 219
8. What are the causes of low (i.e., <1.1g/dL) serum-ascites albumin gradients? 219
9. What are the variants of ascitic fluid infection? 219
10. What is the diagnostic criterion of spontaneous bacterial empyema? 219
11. How do you differentiate spontaneous from secondary peritonitis? 219
12. Who is at high risk of developing SBP? 220
13. What is the pathogenesis of SBP? 220
14. What single test provides early information about possible ascitic fluid infection? 220
15. What is the treatment of choice for suspected SBP? 220
16. When should antibiotic treatment be started in a patient with cirrhosis and suspected ascitic fluid infection? 221
17. Should the PMN cell count in ascitic fluid be monitored during treatment of SBP? 221
18. Does bacterascites represent a real peritoneal infection? Should it be treated? 221
19. What does the presence of bacterial DNA in blood and ascitic fluid represent in cirrhotic patients? 222
20. Which subgroups of patients with liver disease should receive treatment to prevent bacterial infection? 222
21. Are there alternative prophylactic treatments to quinolones for preventing bacterial infections in cirrhosis? 222
22. What is the treatment of spontaneous bacterial empyema? 222
23. Why is it important to know the sodium balance in patients with cirrhosis and ascites? 223
24. Describe the initial treatment of patients with cirrhosis and ascites 223
25. What is refractory ascites? 223
26. Which patients should be treated with large-volume paracentesis? 224
27. Should volume expanders be infused after large-volume paracentesis? 224
28. Is there currently any indication for peritoneovenous shunt? 224
29. Which patients with cirrhosis and ascites should be considered for TIPS? 224
30. Which patients with cirrhosis and ascites should be evaluated for liver transplantation? 224
31. What is the treatment of hepatic hydrothorax? 225
32. What is dilutional hyponatremia in patients with cirrhosis? 225
33. What is the treatment of dilutional hyponatremia? 225
34. What is the hepatorenal syndrome? 225
35. What are the criteria of hepatorenal syndrome? 225
36. Describe the treatment of patients with hepatorenal syndrome 225
37. Is it possible to prevent hepatorenal syndrome? 226
Website 226
Bibliography 226
Chapter 31: Liver Abscess 228
1. What are the two major categories of liver abscess? 228
2. Describe the clinical features of pyogenic liver abscess 228
3. What are the clinical features of amebic liver abscess? 228
4. What laboratory features are distinctive in patients with liver abscess? 228
5. What are the most common sources of pyogenic liver abscess? 228
6. List the organisms that commonly cause pyogenic liver abscess 229
7. Do negative cultures from an abscess aspirate indicate a nonpyogenic abscess? 229
8. What abnormalities can be detected on standard radiologic studies of patients with liver abscess? 229
9. Which imaging studies should be obtained in evaluating a suspected liver abscess? 229
10. What areas of the liver are usually affected by hepatic abscess? 229
11. How can the location, size, and number of liver abscesses help to determine the source? 230
12. When should a hepatic abscess be aspirated? 230
13. In what situation should an amebic liver abscess be treated by open surgical drainage? 230
14. Does aspiration of an amebic hepatic abscess yield diagnostic material in most patients? 230
15. How often is the biliary tree involved in patients with amebic liver abscess? 230
16. How can the diagnosis of an amebic abscess be confirmed? 230
17. Describe the treatment for pyogenic liver abscess 230
18. Describe the treatment for amebic liver abscess 231
19. List the potential complications of pyogenic liver abscess 231
20. List the potential complications of amebic liver abscess 231
21. What is the prognosis for patients with liver abscess? 231
22. Is a vaccine against amebiasis feasible? 231
Website 231
Bibliography 232
Chapter 32: Inheritable Forms Of Liver Disease 233
Hemochromatosis 233
1. How do we classify the various iron-loading disorders in humans? 233
2. What are neonatal iron overload and African iron overload? 233
3. How much iron is usually absorbed per day? 233
4. Where is iron normally found in the body? 233
5. Discuss the genetic defect in patients with HH. 234
6. What are the usual toxic manifestations of iron overload? 234
7. What are the most common symptoms in patients with HH? 234
8. Describe the most common physical findings in patients with HH. 234
9. How is the diagnosis of hemochromatosis established? 234
10. How commonly do abnormal iron studies occur in other types of liver disease? 235
11. Is computed tomography (CT) or magnetic resonance imaging (MRI) useful indiagnosing hemochromatosis? 235
12. On liver biopsy, what is the typical cellular and lobular distribution of iron in HH? 235
13. How useful is HIC? 235
14. How is the HII used in diagnosing HH? 235
15. How do you treat a patient with HH? 235
16. What kind of a response to treatment can you expect? 235
17. What is the prognosis for a patient with hemochromatosis? 236
18. Because hemochromatosis is an inherited disorder, what is my responsibility tofamily members once a patient has been identified? 236
19. Should general population screening be done to evaluate for hemochromatosis? 236
a1-Antitrypsin Deficiency 236
20. What is the function of a1-AT in healthy people? 236
21. How common is a1-AT deficiency? 236
22. Where is the abnormal gene located? 236
23. What is the nature of the defect that causes a1-AT deficiency? 236
24. Describe the common symptoms and physical findings of a1-AT deficiency. 236
25. How is the diagnosis of a1-AT deficiency established? 236
26. What histopathologic stain is used to diagnose a1-AT deficiency? 237
27. How is a1-AT deficiency treated? 237
28. What is the prognosis for patients with a1-AT deficiency? Should family screening beperformed? 237
Wilson Disease 237
29. How common is Wilson disease? 237
30. Where is the Wilson disease gene located? 237
31. What is the usual age of onset of Wilson disease? 237
32. Which organ systems are involved in Wilson disease? 237
33. What are the different types of hepatic manifestations in Wilson disease? 237
34. How is the diagnosis of Wilson disease established? 237
35. What forms of treatment are available for patients with Wilson disease? 238
36. Is it necessary to perform family screening in Wilson disease? 238
37. Compare Wilson disease and HH. 238
Website 238
Bibliography 238
Chapter 33: Liver Histopathology 239
Liver Microanatomy And Injury Patterns 239
1. Explain the role of liver biopsy. 239
2. Many liver biopsy reports say that the basic architecture is intact and then list astring of abnormalities. What is the basic architecture? 239
3. What are the geographic differences in pathology between portions of hepatic acini? 239
4. What is meant by distortion of the hepatic architecture? 239
5. How are degrees of fibrosis designated? 239
6. What criteria are used to define the presence of cirrhosis? 239
7. Can cirrhosis be diagnosed on a needle biopsy specimen? 239
8. What types of liver cell injury are seen on needle specimens? What causes eachtype? 240
Fatty Change And Steatohepatitis 240
9. Injury from either acute or chronic ethanol ingestion is one of the most commoninsults to the liver. Describe the major characteristics of mild and severe injury. 240
10. What are Mallory bodies? 240
11. How does scarring progress with alcoholinjury? 240
12. Is alcoholic cirrhosis micronodular ormacronodular? 240
13. Sometimes a biopsy shows alcoholic hepatitis, but the patient denies drinkingethanol. Is the pathologist’s diagnosis incorrect, or is there a differential diagnosisfor alcoholic hepatitis? 241
Viral Hepatitis 241
14. How can a liver biopsy help in patients with viral hepatitis? 241
15. When, if ever, is a biopsy ordered for patients with hepatitis A? With hepatitisB and C? 241
16. Does chronic hepatitis have unique histopathologic features? 241
17. What features are typical of chronic hepatitis? 241
18. How is chronic hepatitis graded and staged? 241
19. What features in the liver biopsy help to predict etiology? 242
20. Can chronic viral hepatitis be confused with other injuries? 242
Cholestasis 242
21. In patients with acute or chronic cholestasis, can the liver biopsy distinguish amongthe various differential diagnoses? 242
Drug Injury 242
22. What histologic changes suggest drug- or toxin-related liver injury? 242
Bile Duct Disorders 243
23. In a patient with large duct obstruction, conjugated hyperbilirubinemia, anultrasound showing bile duct stones, and clinical cholangitis, what would a biopsyshow? 243
24. When is PBC diagnosed? 243
25. How is PBC staged? 243
26. What are the histologic features of PSC? 243
27. How is PSC staged? 244
28. What are the most common biopsy findings in patients with PSC? 244
Granulomatous Inflammation 244
29. What is a granuloma? 244
30. How common are granulomas in liver biopsies? 244
31. What causes granulomas in the liver? 244
32. In patients with fever of unknown origin, do negative stains for fungi and acid-fastbacilli exclude infection? 244
33. What are the different types of granulomas? Is the distinction of diagnostic use? 244
34. How often are liver granulomas secondary to a drug reaction? 244
Inherited Liver Disease 244
35. What is hematochromatosis? 244
36. Is a liver biopsy necessary to diagnosis genetic hemachromatosis? 245
37. What disorders are problematic in the clinical differential diagnosis ofhemochromatosis? 245
38. What is Wilson disease? Can liver biopsy help to establish the diagnosis? 245
39. What other tests are helpful in patients with Wilson disease? 245
40. What are the features of a1-antitrypsin (a1-AT) deficiency on liver biopsy? 245
41. Is the presence of PAS-positive, diastase-resistant globules diagnostic for a1-ATdeficiency? 246
Neoplasms 246
42. Discuss the role of liver biopsy in diagnosing metastatic neoplasms. 246
43. Discuss the role of biopsy in diagnosing primary liver tumors. 246
44. Can the clinical laboratory help in classifying tumors? 246
Transplantation 246
45. Describe the role of liver biopsy in the evaluation of transplant recipients withabnormal liver function tests (LFTs) in the early postoperative period. 246
46. What are the main histologic features ofacute rejection? 246
47. What criteria help to distinguish recurrent hepatitis C after transplantation fromallograft rejection? 247
48. Describe the role of liver biopsy in the evaluation of abnormal LFTs in the first yearafter transplantation (and beyond). 247
49. How can a liver biopsy help in the evaluation of a bone marrow transplant recipientwith elevated LFTs? 247
Websites 247
Bibliography 247
Chapter 34: Hepatobiliary Cystic Disease 249
1. Describe the five major classes and subtypes of congenital bile duct cysts (Fig. 34-1) 249
2. Describe the typical clinical presentation of a bile duct cyst 249
3. Compare the main features of Caroli disease and Caroli syndrome 249
4. What is the incidence of malignancy within a congenital bile duct cyst? 250
5. Describe the preferred treatment for patients with bile duct cyst disease 250
6. What is the role of cholangiopancreatography in patients with bile duct cyst disease? 250
7. Provide a differential diagnosis for a cystic hepatic lesion 250
8. What is the significance of a simple hepatic cyst? 250
9. Describe the ultrasonographic, CT, and magnetic resonance imaging (MRI) characteristics of a simple hepatic cyst 250
10. What disease commonly is associated with polycystic liver disease (PLD)? 251
11. What are the risk factors for polycystic liver disease in patients with ADPKD? 251
12. Describe the clinical manifestations of complicated polycystic liver disease 251
13. How does the presence of liver cysts affect hepatic function? 251
14. What are the treatment options for patients with symptomatic polycystic liver disease? 251
15. What is echinococcosis? 251
16. Describe the usual life cycle of E. granulosus 252
17. Where and how does E. granulosus infect humans? 252
18. Describe the typical clinical presentation of hepatic cystic echinococcosis 252
19. How is cystic echinococcosis diagnosed? 253
20. What are the treatment options for hepatic cystic echinococcosis? 253
21. What hepatobiliary cystic neoplasm with malignant potential can be mistaken for a simple cyst, polycystic liver disease, or a hydatid cyst? 253
Bibliography 253
Chapter 35: Gallbladder: Stones, Sludge, and Polyps 254
1. How common are gallstones in Western populations? 254
2. What is the natural history of asymptomatic and symptomatic gallstones? 254
3. What are the risk factors for gallstones? 254
4. What are the symptoms of biliary colic? 254
5. What are the three principal factors involved in gallstone formation? 254
6. Name some drugs, medical conditions, or medical therapy associated with gallstone/sludge formation 254
7. What is biliary sludge? 254
8. Can gallbladder sludge cause symptoms or complications? 255
9. What is the risk of gallstones in the obese? Why are gallstones more common with obesity? 255
10. Why is rapid weight loss a risk factor for development of gallstones? Can such gallstones be prevented? 255
11. How do yellow, black, and brown biliary stones differ clinically? 255
12. Discuss complications from the migration of gallstones 255
13. What causes acute cholecystitis in patients with gallstones? 255
14. What are the symptoms of acute cholecystitis? How should patients with acute cholecystitis be treated? 255
15. Should patients with asymptomatic stones be treated? What is the treatment of choice for patients with symptomatic stones? 256
16. What treatment options are available for patients who do not want to undergo cholecystectomy? 256
17. Who is a candidate for oral bile acid dissolution therapy? 256
18. What nonsurgical methods are available for stone removal or destruction? 256
19. How accurate is ultrasonography for detection of cholecystolithiasis? Of choledocholithiasis? 256
20. What is the role of magnetic resonance cholangiopancreatography in diagnosing common bile duct stones? 256
21. What is the role of endoscopic ultrasonography in diagnosing common bile duct stones? 257
22. A 65-year-old woman undergoes transabdominal ultrasonography due to postprandial abdominal pain. A 1-cm polyp is found in the gallbladder. What is the differential diagnosis? 257
23. What is a porcelain gallbladder? 257
24. What is Mirizzi syndrome? 257
25. What is the clinical significance of a low gallbladder ejection fraction? 257
26. Describe the clinical manifestations and treatment of acute acalculous cholecystitis 257
Bibliography 257
Chapter 36: Sphincter of Oddi Dysfunction 259
1. What is the sphincter of Oddi? 259
2. How does the sphincter of Oddi function? 259
3. What is sphincter of Oddi dysfunction (SOD)? 259
4. Describe the pathophysiology of SOD 259
5. Name typical symptoms of SOD 259
6. Who is at risk for SOD? 260
7. How common is SOD? 260
8. What diagnostic evaluation should be considered in a patient presenting with symptoms suggestive of SOD? 260
9. Are there noninvasive tests that may be used to diagnose SOD? 260
10. When should you consider endoscopic retrograde cholangiopancreatography (ERCP) with sphincter of Oddi manometry (SOM)? 260
11. How is SOM performed, and what manometric criteria define SOD? 260
12. Which medications can interfere with SOM pressure measurements? 260
13. What are the possible complications of ERCP with SOM? 260
14. In multivariate analysis, have any preprocedural pharmacologic agents been shown to help reduce post-ERCP pancreatitis in patients with suspected SOD? 261
15. What is the Milwaukee classification? 261
16. Does normal SOM on one occasion rule out SOD? 261
17. Are there medicines to treat SOD? 261
18. How is SOD treated endoscopically? 262
19. What is the clinical response rate of sphincterotomy for treatment of SOD? 262
20. Can pharmacologic agents cause clinical SOD? 262
21. During manometry, which segment (biliary or pancreatic) should be studied? 262
22. In SOD patients, when should the pancreatic duct be stented? 262
23. Why do some patients with documented SOD not respond to biliary sphincter ablation? 262
Websites 263
Bibliography 263
Chapter 37: Acute Pancreatitis 264
1. What are the causes of acute pancreatitis (AP)? 264
2. What are the most common causes of AP? 264
3. Which drugs have been reported to cause AP? 264
4. How is pregnancy associated with AP? 265
5. Which infectious agents have been implicated in causing AP? 265
6. How do parasitic infections caused by C. sinensis and Ascaris lumbricoides cause AP? 265
7. Is there an increased incidence of AP in patients with AIDS? 265
8. How does penetrating or blunt trauma cause AP? 265
9. What is pancreas divisum? Is it associated with an increased incidence of recurrent AP? 265
10. What is the relationship between hypertriglyceridemia and AP? 266
11. What is the relationship between hypercalcemia and AP? 266
12. How is the diagnosis of AP made? 266
13. How does serum amylase compare to serum lipase in the diagnosis of AP? 266
14. What are the causes of hyperamylasemia and hyperlipasemia? 266
15. What are macroamylasemia and macrolipasemia? 267
16. What cause of AP should be suspected in patients who present with normal serum amylase levels? 267
17. Does the magnitude of hyperamylasemia or hyperlipasemia correlate with the severity of AP? 267
18. What is the most reliable serum marker for diagnosing biliary AP? 267
19. How is AP classified? 267
20. What prognostic scoring systems are used to assess the severity of AP? 268
21. What is the role of serum markers in assessing the severity of AP? 269
22. What are other prognostic indicators in AP? 269
23. What are the major systemic complications of AP? 269
24. When is infection of pancreatic necrosis suspected? 270
25. What is the most common organism isolated in infected pancreatic necrosis? 270
26. How is AP treated? 270
27. When and via what route should nutritional support be initiated in patients with AP? 270
28. When should ERCP be performed in biliary AP? 271
29. Should patients undergo a cholecystectomy after an episode of biliary AP? 271
30. How soon should a cholecystectomy be performed after an attack of biliary AP? 271
31. Should patients with coexisting alcoholism and cholelithiasis undergo cholecystectomy to prevent further attacks of AP? 271
32. What are acute pancreatic fluid collections? 271
33. What are pseudocysts? 271
34. When should a pseudocyst be suspected? 272
35. What are the indications for pseudocyst drainage? 272
36. How are pancreatic pseudocysts drained? 272
37. What are possible complications of an untreated pancreatic pseudocyst? 272
38. What is a pancreatic abscess? 272
Bibliography 272
Chapter 38: Chronic Pancreatitis 274
1. What classification system is used for chronic pancreatitis (CP)? 274
2. What is the most common cause of CP in adults? 274
3. What are other causes of CP? 274
4. What is autoimmune pancreatitis? 274
5. What is tropical or nutritional pancreatitis? 275
6. What is obstructive CP? 275
7. What is hereditary pancreatitis? 275
8. How is cystic fibrosis associated with CP? 275
9. What is idiopathic CP? 275
10. What is the most common presenting symptom of CP? 275
11. What are the causes of weight loss in patients with CP? 275
12. Is steatorrhea an early symptom of CP? 275
13. Is diabetes mellitus an early manifestation of CP? 275
14. Are measurements of serum pancreatic enzymes helpful in the diagnosis of CP? 276
15. What do elevated levels of bilirubin and alkaline phosphatase suggest in the patient with CP? 276
16. What specialized test directly measures pancreatic exocrine function? 276
17. What conditions may be associated with a false-positive secretin stimulation test? 276
18. What indirect tests of pancreatic exocrine function are used? 276
19. Are plain abdominal radiographs helpful in the diagnosis of CP? 276
20. What other imaging modalities are used in the diagnosis of CP? 276
21. What is the role of endoscopic retrograde cholangiopancreatography (ERCP) in the diagnosis of CP? 277
22. What is the Cambridge Grading system of CP based on ERCP findings? 277
23. What is the role of endoscopic ultrasound (EUS) in the diagnosis of CP? 277
24. What are the EUS criteria for the diagnosis of CP? 277
25. What is the role of MRCP in the diagnosis of CP? 277
26. What is the most common complication of CP? 278
27. How are pseudocysts treated? 278
28. What are other complications of CP? 278
29. How is distal CBD obstruction diagnosed and treated? 278
30. How is duodenal obstruction diagnosed and treated? 278
31. How are pancreatic fistulas treated? 279
32. How is pancreatic ascites or pancreatic pleural effusion diagnosed? 279
33. Why does the presence of gastric varices in the absence of esophageal varices suggest CP? 279
34. Are signs of fat-soluble vitamin deficiencies highly suggestive of CP? 279
35. Are patients with CP predisposed to nephrolithiasis? 279
36. How should hyperoxaluria be treated in patients with CP? 279
37. Can patients with CP develop vitamin B12 malabsorption? 279
38. How is steatorrhea from CP treated? 279
39. What are nonsurgical modalities of pain control in CP? 280
40. Does endoscopy have a role in pain control in CP? 280
41. What is the role of surgery in pain control in CP? 280
Websites 280
Bibliography 281
Chapter 39: Pancreatic Cancer 282
1. How common is pancreatic cancer (PC) and what are the most common types of malignant tumors? 282
2. What are the common symptoms of PC, and where are the tumors usually located? 282
3. What is the Courvoisier sign? 282
4. What is the survival rate for patients with PC? 282
5. What are the identifiable risk factors for PC? 282
6. What genetic alterations have been identified to be associated with increased risk for PC? 283
7. What are the available serum markers for early detection of PC? 283
8. Are there precursor lesions to PC? 284
9. What imaging modalities are used to diagnose PC? 284
10. What is the double-duct sign in patients with PC? 284
11. Are there high-risk groups for the development of PC that may benefit from CT and or EUS screening? 284
12. What are the new staging modalities for PC? 285
13. What are the common biochemical abnormalities in patients with PC? 285
14. Is chemotherapy effective for patients with advanced PC? 285
15. What is the median survival after the diagnosis of advanced PC? 285
16. Describe the role of celiac blockade in patients with PC. 285
17. What is a Whipple resection? 285
18. What surgical procedures are used for cancer in the body and tail of the pancreas? 285
19. When do patients with PC need palliative procedures? 285
Websites 286
Bibliography 286
Chapter 40: Cystic Disease of the Pancreas 287
1. Provide a differential diagnosis for a cystic pancreatic lesion. 287
2. What is the difference between a true pancreatic cyst and a pancreatic pseudocyst? 287
3. Define an acute fluid collection 287
4. Define an acute pancreatic pseudocyst 287
5. Define a chronic pancreatic pseudocyst 287
6. Describe the typical clinical presentation of a pancreatic pseudocyst 287
7. What criteria suggest that a pseudocyst will not resolve spontaneously? 287
8. When should a pseudocyst be drained? 287
9. Compare the three methods for draining a pancreatic pseudocyst 288
10. What criteria suggest that a pancreatic pseudocyst may undergo successful endoscopic drainage? 288
11. Describe a pancreatic abscess 288
12. What clinical criteria suggest the development of a pancreatic abscess? 288
13. Define hemosuccus pancreaticus 288
14. When should you suspect that a cystic pancreatic lesion is not a pseudocyst? 288
15. What is a serous cystadenoma? 288
16. What disease commonly manifests by retinal and central nervous system (CNS) hemangioblastomas, renal cell carcinoma, pheochromocytoma, and pancreatic cysts? 289
17. Describe the characteristics of an MCN 289
18. What is an IPMN? How does it differ from a MCN? 289
19. How does the surgical management of an IPMN differ from that of a MCN? 289
20. What is the utility of EUS in the evaluation of a cystic pancreatic lesion? 289
21. What conditions are most commonly associated with a pancreatic retention cyst? 289
Websites 290
Bibliography 290
Chapter 41: Celiac Disease, Tropical Sprue, Whipple Disease, Lymphangiectasia, Immunoproliferative Small Intestinal Disease, and Nonsteroidal AAnti-Inflammatory Drugs 291
1. What is the best screening test for fat malabsorption? 291
2. What is the best quantitative test for fat malabsorption? 291
3. Under what physiologic conditions is fecal fat excretion increased? 291
4. What is the gluten-sensitive enteropathy (GSE) panel? 291
5. What is tissue transglutaminase? 292
6. Name the conditions to consider in previously responsive patients with celiac sprue who begin to deteriorate (Fig. 41-2) 292
7. What are the hepatic manifestations of celiac sprue, and how are they managed? 292
8. Describe the manifestations of Whipple disease 292
9. What is the differential diagnosis of a macrophage infiltrate of the small bowel lamina propria? 293
10. What causes Whipple disease? 293
11. What are the complications of the enteropathy induced by nonsteroidal anti-inflammatory drugs (NSAIDs)? 293
12. Does scleroderma produce any manifestations in the small bowel? 293
13. How does octreotide affect intestinal motility and bacterial overgrowth in scleroderma? 293
14. Describe the different forms of lymphangiectasia 293
15. What are the clinical manifestations of abetalipoproteinemia? 293
16. What are the different clinical presentations of eosinophilic gastroenteritis? 293
17. How are patients with eosinophilic gastroenteritis treated? 294
18. What are the common causes of diarrhea in a patient with Crohn’s disease and ileal resection? 294
19. Where are the endemic areas for tropical sprue? 294
20. How is tropical sprue treated? 294
21. How is bacterial overgrowth diagnosed? 294
22. What is the mechanism of hyperoxaluria in short bowel syndrome? 294
23. What is immunoproliferative small intestinal disease (IPSID)? 294
24. What causes IPSID and how is it treated? 295
25. What are the most common clinical manifestations of IPSID? 295
26. What is video capsule endoscopy, and what are the indications/contraindications for its use? 295
Websites 295
Bibliography 295
Chapter 42: Crohn’S Disease 297
Diagnosis 297
1. What are the usual symptoms and signs suggestive of Crohn’s disease? 297
2. How is the diagnosis of Crohn’s disease established? 297
3. Which diseases can mimic the symptoms and signs of Crohn’s disease? 298
4. What serologic tests can help established the diagnosis? 298
Etiology 299
5. Is cigarette smoking associated with Crohn’s disease? 299
6. What infectious agents might be responsible for Crohn’s disease? 299
7. Is there a genetic predisposition for developing Crohn’s disease? 299
Natural History 299
8. Is mortality increased in patients with Crohn’s disease? 299
9. Are there factors that predict a flare-up of Crohn’s disease activity? 299
10. Does behavior of disease predict its natural history? 299
11. Do patients with Crohn’s disease have an excess cancer risk? 300
12. What are the extraintestinal manifestations of Crohn’s disease? 300
Treatment 300
13. Which 5-aminosalicylic acid preparations are effective in treating Crohn’s diseasepatients? 300
14. Should steroids be used in Crohn’s disease? 300
15. What is the role for immunosuppressive therapy in Crohn’s disease? 300
16. Which biologic therapies are effective for patients with Crohn’s disease? 301
17. Which medications are effective in maintaining remission? 301
18. What are the indications for surgery in Crohn’s disease? 301
19. What therapeutic regimen is most often effective for stricturing-type Crohn’sdisease? 302
20. What therapeutic regimen is most often effective for inflammatory-type Crohn’sdisease? 302
21. What therapeutic regimen is most often effective for fistulizing Crohn’s disease? 302
22. When should nutritional supportbe used in patients with Crohn’sdisease? 302
Website 302
Bibliography 302
Chapter 43: Ulcerative Colitis 304
1. What is ulcerative colitis (UC)? 304
2. Define backwash ileitis 304
3. What is indeterminate colitis? 304
4. Why is it important to distinguish between UC and Crohn’s disease? 304
5. What causes UC? 304
6. Who gets UC? 304
7. What are the signs and symptoms of UC? 304
8. How are patients with UC classified? 305
9. How are the extraintestinal manifestations of UC classified? 305
10. What is colitic arthritis? 305
11. Describe the association between UC and ankylosing spondylitis 305
12. Discuss the hepatic complications of UC 305
13. What are the ocular complications of UC? 306
14. Describe the association between UC and thromboembolic events 306
15. How do I evaluate a patient with UC? 306
16. What are 5-ASA products? 306
17. How do I treat proctitis and proctosigmoiditis? 306
18. How do I treat an exacerbation of UC? 307
19. What should I do if the disease is severe? 307
20. Define toxic megacolon 307
21. How do I prevent a relapse? 307
22. Are there adjunctive therapies for UC? 308
23. How often should patients have surveillance colonoscopy? 308
24. What should be done if a polyp or dysplasia is found? 308
25. Is surveillance effective? 308
26. Is there a role for chemoprevention in UC? 308
27. Is diet important in the management of UC? 309
28. Does stress exacerbate UC? 309
29. How does menstruation affect UC? 309
30. Do patients with UC have problems with fertility and pregnancy? 309
31. What medications are contraindicated in patients with UC? 309
32. What are the surgical options for management of UC? 309
Websites 310
Bibliography 310
Chapter 44: Eosinophilic Gastroenteritis 311
1. How is eosinophilic gastroenteritis (EGE) defined? 311
2. What is the incidence of EGE? 311
3. What is the etiology of EGE? 311
4. What are the clinical features of EGE? 311
5. What is the Klein classification for EGE? 311
6. Why does EGE have so many different clinical faces? 311
7. Is there an increase in the recognition of cases of eosinophilic infiltration of the GI tract? 313
8. What is hypereosinophilia syndrome (HES)? 313
9. What is the differential diagnosis of EGE? 314
10. What are possible radiographic features of EGE? 315
11. What are rational steps to diagnose EGE? 315
Laboratory Studies 315
12. What should be excluded in patients with suspected EGE? 316
13. What are other treatment modalities for EGE? 316
14. What is the natural history of EGE? 316
Websites 317
Bibliography 317
Chapter 45: Bacterial Overgrowth 318
1. Define bacterial overgrowth 318
2. What is the usual bacterial presence in the gastrointestinal tract? 318
3. What are the natural protective mechanisms against SIBO? 318
4. What factors influence small intestinal bacterial proliferation? 318
5. What kind of structural lesions predispose to overgrowth? 318
6. How do motility disorders cause overgrowth? 318
7. How can an excessive bacterial load be delivered to the small bowel? 318
8. Which impairments of host defenses are important? 318
9. What conditions are associated with small intestinal bacterial overgrowth? 319
10. What are the other risk factors for bacterial overgrowth? 319
11. What are the symptoms of overgrowth? 319
12. Why do patients with bacterial overgrowth develop anemia? 319
13. What other micronutrient deficiencies are clinically important? 319
14. What is the association between SIBO and sepsis? 319
15. How is bacterial overgrowth diagnosed? 320
16. What indirect testing can be used? 320
17. What are the limitations of testing? 320
18. What about other testing methods? 320
19. What is the treatment for bacterial overgrowth? 320
20. What are the antibiotic agents used in the treatment of SIBO? 321
21. Do prokinetic agents help? 321
22. How long should overgrowth be treated with antibiotics? 321
23. What about probiotics? 321
Bibliography 321
Chapter 46: Colorectal Cancer And Colon Cancer Screening 323
1. What is colorectal cancer (CRC)? 323
2. How does the pathophysiology of rectal cancer differ from cancer elsewhere in the colon? 323
3. How common is colorectal cancer? 323
4. Do the genetic defects leading to sporadic CRC differ from those in genetic syndromes associated with colon cancer? 323
5. Describe the natural sequence from colon adenoma to colon cancer 323
6. How prevalent are colonic adenomas among the U.S. population? 324
7. Where in the colon are polyps most commonly located? 324
8. Give the mean age of onset and describe the anatomic distribution of CRC 324
9. How are malignant polyps defined? How are they clinically managed? 324
10. How is colon cancer staged, and how does this affect prognosis? 324
11. Describe the workup for CRC after the initial diagnosis 325
12. What surgical margins are recommended? 325
13. Describe the recommended schedule of colonoscopic follow-up after surgery 325
14. Are there any effective blood tests to screen for CRC? 325
15. List the risk factors for developing CRC 325
16. What is the effect of age on the risk of developing CRC? 325
17. Discuss the effect of diet on the risk for developing CRC 325
18. Do environmental factors increase the risk for developing CRC? 326
19. Which adenoma features are associated with a greater malignant potential? 326
20. What are the recommendations for CRC screening in people at average risk for CRC? 326
21. Give the current guidelines for surveillance colonoscopy in patients with a history of adenomatous polyps 326
22. Who is considered to be at increased risk for developing CRC? 326
23. Which method of CRC screening is recommended for individuals at increased risk of developing CRC? 326
24. List the familial colon cancer syndromes 327
25. What tests are available for hereditary CRC? 327
26. What is the recommended surveillance for people with a family history of CRC who do not fit the genetic profiles? 327
27. How do FAP and Gardner syndrome increase the risk of CRC? 328
28. How are FAP and Gardner syndrome diagnosed? 328
29. In addition to colonoscopy, what other tests should be considered in FAP? 328
30. What is the role of NSAIDs in treating FAP? 328
31. How do hamartomatous polyp syndromes affect the risk of developing CRC? 328
32. What is HNPCC? 328
33. How is HNPCC diagnosed? 328
34. Outline the screening recommendation for patients with HNPCC 329
35. What is MYH-associated polyposis (MAP)? 329
36. What is microsatellite instability (MSI)? 329
37. How does inflammatory bowel disease (IBD) affect the risk of developing CRC? 329
38. Which two clinical conditions should raise suspicion for the presence of colon cancer? 329
39. Is FOBT effective in detecting colon polyps and cancer? 329
40. Does a program of periodic sigmoidoscopy decrease mortality from CRC? 330
41. What is the sensitivity and specificity of an air contrast barium enema? 330
42. How effective is CT colonography (CTC) as a screening test? 330
43. Can CRC be prevented with medicines (chemoprevention)? 330
Bibliography 330
Chapter 47: Constipation And Fecal Incontinence 332
1. What is constipation? 332
2. Describe the normal mechanism of stool passage 332
3. What are the major causes of constipation? 333
4. Describe the workup for constipation 333
5. What tests are used in the evaluation of chronic constipation? 333
6. How can primary constipation be further defined? 334
7. What causes impaired colonic transit? How is it diagnosed? 334
8. What causes impaired rectoanal inhibitory reflex? How is it diagnosed? 334
9. What is dyschezia? 334
10. What is anorectal dyssyngergia? 334
11. What other physiologic abnormalities may lead to anorectal dysfunction, and how are they diagnosed? 335
12. How is constipation due to irritable bowel syndrome diagnosed? 335
13. Describe the general management of constipation 335
14. Describe the proper use of dietary fiber 335
15. How do osmotic laxatives work? 336
16. How do cathartics work, and are stool softeners effective? 336
17. What is lubiprostone, and how does it work? 336
18. Is surgery ever indicated for constipation? 336
19. Are there other treatments for constipation? 336
20. What is fecal incontinence? Who is generally affected? 336
21. Describe the pathophysiology of fecal incontinence 337
22. What are the risk factors associated with incontinence? 337
23. Describe the workup for incontinence 337
24. What specialized tests are available for the evaluation of incontinence? 338
25. What is medical therapy for incontinence? 338
26. What is biofeedback and how is it used to treat incontinence? 338
27. What are surgical options for treating incontinence? 339
Website 339
Bibliography 339
Chapter 48: Diverticulitis 340
1. What is a diverticulum? What type are colonic diverticula? 340
2. How common is diverticular disease? What are the most frequent complications? 340
3. How do diverticula develop? Who is at risk for developing diverticulosis? 340
4. What is hypersegmentation? 340
5. What is myochosis? 340
6. Where are diverticula located? 340
7. How does diverticulitis develop? 340
8. How should symptomatic diverticulosis be managed? 341
9. What is diverticular or segmental colitis? 341
10. What are the common signs and symptoms of early diverticulitis? 341
11. What are the signs and symptoms of severe diverticulitis? 341
12. What is the Hinchey classification system? How does it predict outcome? 341
13. What is the natural history of diverticulitis? 341
14. List the common complications of diverticulitis. 342
15. Between what organs do fistulous communications develop? 342
16. What techniques are used to diagnose and localize fistulas? 342
17. How is a diverticular stricture differentiated from strictures of other causes? 342
18. Which drugs are known to exacerbate diverticulitis? 342
19. What imaging modalities are available to diagnose diverticulitis? What is the role of each? 342
20. How is mild diverticulitis defined and treated? 342
21. What antibiotic regimen is appropriate for moderately severe disease? How is treatment otherwise different? 343
22. How is the management of severely ill patients different? 343
23. What are the indications and goals for surgery? 343
24. What operations are available in the management of diverticulitis? 344
Websites 345
Bibliography 345
Chapter 49: Diseases Of The Appendix 346
1. Describe the anatomy and function of the human appendix 346
2. What is the presumed etiology of appendicitis? 346
3. What are the signs and symptoms of appendicitis? 346
4. What are the laboratory findings? 346
5. Where and what is the McBurney point? 346
6. What are the psoas and obturator signs? 346
7. What is the Rovsing sign? 346
8. The peak incidence of acute appendicitis occurs in what age group? 346
9. The risk of perforation of the appendix is highest in what age groups? 346
10. What is the surgical mortality rate for nonperforated appendicitis? Perforated appendicitis? 346
11. List the differential diagnosis for right lower quadrant pain both in women and in children. 347
12. What is a Meckel’s diverticulum? 347
13. What is an acceptable incidence rate for negative appendectomy? Has this rate changed with the increasing use of ultra ... 347
14. In older patients (older than 50 years), what condition may be indistinguishable from acute appendicitis? 347
15. What features of pelvis inflammatory disease (PID) can help distinguish it from appendicitis? 347
16. What is the most common malignant tumor of the appendix? Describe its management 347
17. What is the proper treatment for late/perforated appendicitis that presents as an abscess? 347
18. What is the most common complication after appendectomy? 347
19. In what patient population is ultrasound particularly helpful in making the diagnosis of acute appendicitis? 347
20. What other imaging modality is often used (and abused)? 348
21. When is laparoscopic appendectomy appropriate? 348
22. During an abdominal exploration for right lower quadrant pain, is removal of a normal appendix appropriate in patients with Crohn’s disease? 348
23. Is an appendectomy during pregnancy a safe procedure? 348
24. If an ovarian tumor is discovered during laparoscopic or open exploration, what steps should be taken? 348
25. Does nonoperative therapy have any role in treating acute appendicitis? 348
26. What is a Mitrofanoff procedure? 348
Bibliography 348
Chapter 50: Colitis: Pseudomembranous, Microscopic, and Radiation 349
Pseudomembranous Colitis 349
1. What is Clostridium difficile? 349
2. How is CDI defined? 349
3. What causes PMC? 349
4. What are the risk factors for CDI? 349
5. Which antibiotics are most commonly implicated? 349
6. Why do some people develop C. difficile diarrhea while others are simply colonized? 349
7. How has the epidemiology of CDI changed over the past decade? 350
8. What accounts for the changing epidemiology of CDIs? 350
9. What possible factors mediate the hypervirulence of the BI/NAP1/027 strain? 350
10. How is the diagnosis of CDI made? 350
11. What are the typical findings on colonoscopy? 350
12. What are the hallmarks of severe CDI? 351
13. When is treatment indicated? What antibiotics are used? 351
14. When should you expect a response to treatment? 351
15. What other treatment options are under development and investigation? 351
16. What should you do if symptoms recur after therapy? 351
17. How can we control C. difficile epidemics in hospitals? 351
Microscopic Colitis 352
18. What is microscopic colitis (MC)? 352
19. What are the features of CC and LC? 352
20. What are the clinical features of MC? 352
21. How do you distinguish MC from patients with irritable bowel syndrome (IBS)? 352
22. Are there any laboratory tests or imaging studies that can help establish thediagnosis of MC? 352
23. How common is MC? 352
24. Which parts of the colon are most commonly affected? 352
25. What agents are associated with the pathogenesis of MC? 353
26. What are the associated conditions? 353
27. What is the natural history of MC? 353
28. What are the treatment options? 353
Radiation Colitis 353
29. Which part of the gastrointestinal tract is most commonly injured by radiation? 353
30. What can be done to prevent radiation damage? 353
31. What symptoms are associated with irradiation? 353
32. What are effects of localized radiation to the colon? 353
33. How can radiation colitis and proctitis be managed? 353
34. What are the endoscopic therapies for chronic bleeding? 353
35. How are chronic radiation-induced bowel strictures managed? 354
Bibliography 354
Chapter 51: Upper Gastrointestinal Tract Hemorrhage 355
1. What are the signs and symptoms of upper gastrointestinal (UGI) bleeding? 355
2. What historic facts will help with determining the source of UGI bleeding? 355
3. How can the amount of acute blood lost be estimated clinically? 355
4. How might one distinguish an UGI bleed from a lower GI bleed in a patient who presents with blood per rectum? 355
5. What are the first steps in managing a patient with UGI bleeding? 355
6. How does one interpret the Hct values in a patient with acute UGI bleeding? 355
7. Why place a nasogastric (NG) tube? 355
8. What types of fluid should be used for resuscitation and when? 356
9. Does every patient with UGI bleeding need to be hospitalized in the intensive care unit or even hospitalized? 356
10. What are the common causes of UGI bleeding? And uncommon causes? 356
11. What are the endoscopic stigmata of bleeding peptic ulcer? How do they help stratify risk for rebleeding and mortality 356
12. What is the role of NSAIDs in UGI bleeding? 358
13. How can one preventing bleeding in patients taking NSAIDs? 358
14. What are the possible sources of bleeding in a patient with cirrhosis who presents with UGI bleeding? 359
15. How does one diagnose bleeding from a varix? 359
16. Which patients need endoscopy and when? 359
17. What techniques are available to the endoscopist for controlling active bleeding? 359
18. What nonendoscopic therapies can be used to stop variceal bleeding? 359
19. What endoscopic therapy is available to control variceal hemorrhage? 359
20. What are the special considerations that need to be addressed in patients with cirrhosis who have acute UGI bleeding? 359
21. What is a visible vessel, and what is its significance? 359
22. Is there a role for other diagnostic tests when evaluating patients with UGI bleeding? 360
23. Which medications, if any, can be used to reduce rebleeding from UGI tract ulcers? 360
24. When and who should be treated with surgery for continued nonvariceal UGI bleeding? 361
25. What medications are used for patients who go home after a bleeding episode? 361
26. When and what should patients receive by mouth after an UGI bleed? 361
27. When should patients be sent home after a UGI bleed? 361
28. What should patients avoid once they have had a UGI bleed? 361
29. How and when should patients be followed up after their episode of UGI bleeding? 361
Websites 361
Bibliography 361
Chapter 52: Lower Gastrointestinal Tract Bleeding 363
1. Define lower gastrointestinal bleeding (LGIB) 363
2. How common is LGIB? 363
3. What populations are at increased risk? 363
4. How does the risk of LGIB compare to that of upper GI bleed (UGIB)? 363
5. What is the mortality associated with LGIB? 363
6. How is history important in assessing a patient with LGIB? 363
7. What can help differentiate between an upper and a lower source of bleeding? 363
8. What are the first steps taken in the management of a patient with significant LGIB? 364
9. How can continued or recurrent LGIB be determined? 364
10. What are the causes of most common causes of LGIB? 364
11. Do NSAIDs increase the risk of LGIB? 365
12. What types of colitis are associated with LGI bleeding? 365
13. Do all colonic vascular ectasias or angiodysplasia cause LGIB? 365
14.How is postpolypectomy LGIB best managed? 365
15. What diagnostic modalities are available for localization of colonic bleeding? 365
16. What role does urgent colonoscopy have in the diagnosis of LGIB? 365
17. Discuss how nuclear medicine scintigraphy and angiography are used in the diagnosis and treatment of LGIB 365
18. What is the natural history of LGI bleeding from diverticulosis? 366
19. What endoscopic methods are available for hemostasis? 366
20. What are the more common causes of small intestinal bleeding? 366
21. What diagnostic modalities are available for small intestinal bleeding? 366
22. How does double-balloon endoscopy (DBE) compare with video capsule endoscopy? 366
23. What is the role of surgery in LGIB? 367
Websites 367
Bibliography 367
Chapter 53: Occult and Obscure Gastrointestinal Bleeding 368
1. What is occult gastrointestinal (GI) bleeding? 368
2. What physical examination findings might provide a clue about the source of bleeding? 368
3. What tests are used to identify patients with occult GI bleeding, and which are the best? 368
4. What are the factors that influence the results of FOB testing besides bleeding from the GI tract? 368
5. What is the proper procedure for FOB testing? 368
6. How much blood is needed to cause a positive FOB test? 368
7. Who should be electively tested for occult blood and how often? 368
8. What can be expected to be found at colonoscopy in a patient older than 50 years who is FOB negative? Who is FOB positive? 368
9. How should a patient with a positive FOB test be evaluated? 369
10. What are some of the signs and symptoms of iron deficiency anemia? 369
11. How should a patient with a positive FOB and iron deficiency anemia be initially evaluated? 369
12. What tests would you do in a patient with iron deficiency (microcytic) anemia who does not respond to iron or has recurrence after an initial negative evaluation? 369
13. How would the evaluation be different if the patient was only iron deficient? 369
14. What is the yield for combined colonoscopy and EGD in patients who are FOB positive with or without iron deficiency? 369
15. How is sprue (celiac disease) diagnosed? 369
16. What is meant by obscure GI bleeding? 369
17. What endoscopic tests are available for evaluating a patient with obscure GI bleeding, and how useful are they? 369
18. What radiologic tests are available for evaluating a patient with obscure GI bleeding, and how useful are they? 370
19. How would you sequence an evaluation of a patient with obscure GI bleeding? 370
20. Angiodysplasia (vascular malformations) are a common cause of obscure GI bleeding. How are these treated? 370
21. What other lesions are found to be a cause of obscure GI bleeding? 370
Websites 371
Bibliography 371
Chapter 54: Evaluation of Acute Abdominal Pain 372
1. Provide a useful clinical definition of an acute abdomen 372
2. What are the four types of stimuli for abdominal pain? 372
3. What are the three categories of abdominal pain? 372
4. How does the character of the abdominal pain help in the evaluation? 372
5. What are the important components of the physical examination for patients with acute abdominal pain? 372
6. Which laboratory tests should be obtained in patients with acute abdominal pain? 373
7. Which radiologic tests should be ordered to evaluate the patient with acute abdominal pain? 373
8. Pain referred to the abdomen can be confusing. What are the common extra-abdominal causes of referred abdominal pain? 373
9. List the common causes of acute abdominal pain in gravid women 374
10. When the appendix is found to be entirely normal during a laparotomy performed for presumed appendicitis in a gravid woman, should the appendix be removed? 374
11. What is the most common cause of acute abdominal pain in elderly patients? 374
12. What symptoms are helpful in evaluating for appendicitis? 374
13. Discuss atypical forms of appendicitis 374
14. Describe the ultrasound findings of acute appendicitis 374
15. When laparotomy is performed for presumed appendicitis, what is the acceptable false-negative rate? How often is another cause identified in this setting? 374
16. What is the single best test to evaluate patients infected with human immunodeficiency virus (HIV) infection who complain of acute abdominal pain? 375
17. What are the cardinal features of a ruptured tubal pregnancy? 375
18. What are the characteristics of acute intestinal obstruction? 375
19. List the clinical characteristics of large bowel obstruction 375
20. List the clinical characteristics of diverticulitis 375
21. What are the characteristic CT findings of diverticulitis? 375
22. List the clinical hallmarks of acute cholecystitis 375
23. What is the differential diagnosis of acute cholecystitis? 375
24. When should a patient undergo surgery for an acute abdomen? 376
25. What conditions can result in an acute abdomen in HIV-infected patients? 376
26. Are patients with systemic lupus erythematosus (SLE) at increased risk for intra-abdominal catastrophe? 376
27. How common are severe GI manifestations of polyarteritis nodosa (PAN)? 376
28. What causes of acute abdominal pain should be considered in illicit drug users? 376
Websites 376
Bibliography 376
Chapter 55: Evaluation of Acute Diarrhea 378
1. What is the definition of acute diarrhea? 378
2. What is the impact of acute diarrhea in the United States and worldwide? 378
3. Who should undergo medical evaluation for acute diarrhea? 378
4. What are the most common causes of acute bloody diarrhea? 378
5. What is dysentery? 378
6. Name the common causes of infectious dysentery in the United States 378
7. What is the significance of stool leukocytes (white blood cells) and how are they detected? 378
8. If 100 random patients with acute diarrhea underwent evaluation with stool cultures, how many would be positive? Which patients with acute diarrhea should be evaluated with a stool culture? 378
9. Which patients with acute diarrhea should be evaluated with an endoscopic examination? 379
10. By what mechanisms do toxigenic organisms produce diarrhea? 379
11. Which Campylobacter sp. are implicated as causes of dysentery? How is Campylobacter transmitted? 379
12. Describe the clinical and endoscopic features of Campylobacter diarrhea 379
13. How are Salmonella organisms classified? 379
14. How is Salmonella infection acquired? 379
15. List the types of illnesses that can be caused by Salmonella 379
16. What is typhoid fever? 379
17. How is Salmonella infection treated? 380
18. Describe the characteristics of Shigella infection. How is it treated? 380
19. What diarrheogenic illnesses are caused by E. coli? 380
20. What is the therapy for O157:H7-induced diarrhea? 381
21. Describe the clinical presentation of infection with Yersinia enterocolitica 381
22. Which organisms are associated with seafood-induced diarrhea? 381
23. What parasites cause bloody diarrhea? 381
24. Who is at risk for amebiasis? What are the potential complications of amebic dysentery? 381
25. Which laboratory studies are useful in the diagnosis of amebic dysentery? 381
26. Describe the treatment of amebic dysentery. What are the potential side effects? 381
27. Which parasites typically cause nonbloody diarrhea? What are the risks for acquisition? 381
28. What is the most common cause of hospital-acquired diarrhea? 382
29. List the risk factors and therapy for infectious dysentery 382
30. The use of empiric antibiotics in the treatment of acute diarrhea is potentially detrimental in what ways? 382
31. Are antimotility agents contraindicated in patients with dysentery? 382
32. Several members of a family develop nausea, emesis, and watery diarrhea 2 to 6 hours after a picnic. Food at the picnic included ham, rice, and custard pie. What type of bacteria is likely to be the cause? 383
33. What are the common causes and incidence of travelers’ diarrhea? 383
34. How can one avoid travelers’ diarrhea? 383
35. Describe the treatment of travelers’ diarrhea 383
36. What is cholera? 383
37. How is cholera treated? 383
38. What is oral rehydration solution? How does it work? 383
39. What is a BRAT diet? 383
40. What viruses cause acute diarrhea? 384
41. What are the clinical features of rotavirus gastroenteritis? What tests are available for diagnosis? 384
42. You are on your honeymoon cruise, and 25% (300 people) of the ship’s occupants are afflicted with acute gastroenteritis. What is the most likely causativeagent? 384
43. A 42-year-old woman is experiencing lower abdominal cramping, bloating, and intermittent diarrhea 6 months following an episode of dysentery that she experienced during a trip to Mexico. What are the possible mechanisms ofher illness? 384
44. What is Reiter’s syndrome? Which enteric infections are associated with its development? 384
45. What is toxic megacolon? What are its risk factors? 384
46. How does one differentiate between acute infectious dysentery and acute onset of inflammatory bowel disease as the cause of bloody diarrhea? 384
47. How is acute bacterial dysentery differentiated from acute onset of ischemic colitis? 384
Website 385
Bibliography 385
Chapter 56: Chronic Diarrhea 386
1. Define chronic diarrhea 386
2. What other disorder may be described as diarrhea? 386
3. What is the basic mechanism of all diarrheal diseases? 386
4. What pathologic processes can cause diarrhea? 386
5. List three classifications of diarrheal diseases 387
6. What are the likely causes of diarrhea, according to epidemiologic characteristics? 387
7. What are the likely causes of osmotic watery diarrhea? 388
8. List the likely causes of secretory watery diarrhea 388
9. List the likely causes of inflammatory diarrhea 388
10. List the likely causes of fatty diarrhea 389
11. Summarize the initial diagnostic scheme for patients with chronic diarrhea? 389
12. How do you distinguish secretory and osmotic watery diarrhea? 389
13. How is the fecal osmotic gap calculated? 390
14. How is the fecal osmotic gap interpreted? 390
15. What precautions are necessary when measuring fecal osmotic gaps? 390
16. How does one evaluate osmotic diarrhea? 390
17. Describe the evaluation of chronic secretory diarrhea 390
18. When should neuroendocrine tumors be suspected as a cause of chronic secretory diarrhea? 391
19. What is Bayes theorem? How does it relate to the diagnosis of peptide-secreting tumors? 391
20. What is the likely outcome in patients with chronic secretory diarrhea in whom a diagnosis cannot be reached? 392
21. Describe the evaluation of chronic fatty diarrhea 392
22. How does one make a diagnosis of celiac disease? 392
23. Describe the further evaluation of chronic inflammatory diarrhea 392
24. How does one distinguish irritable bowel syndrome from chronic diarrhea? 392
25. What causes of chronic diarrhea may be difficult to diagnose? 393
26. What are common causes of iatrogenic diarrhea? 393
27. What features should suggest surreptitious laxative ingestion? 393
28. What is microscopic colitis syndrome? 394
29. Define bile acid diarrhea 394
30. What is the likely outcome in chronic idiopathic secretory diarrhea? 394
31. What is the best nonspecific therapy for chronic diarrhea? 394
Websites 395
Bibliography 395
Chapter 57: Aids and the Gastrointestinal Tract 396
1. What is the role of barium esophagram for patients with AIDS (acquired immunodeficiency syndrome) and esophageal symptoms? 396
2. What is the role of empiric therapy for new-onset esophageal symptoms in patients with AIDS? 396
3. What are the most common causes of esophageal ulceration in AIDS? 396
4. What biopsy technique should be used to sample an esophageal ulcer? 396
5. What is AIDS-cholangiopathy? How do patients present? 397
6. What are the most common causes of AIDS-cholangiopathy? How are they diagnosed? 397
7. What are the most common causes of pancreatitis in HIV-infected patients? 397
8. How has HAART affected the incidence of opportunistic gastrointestinal (GI) disorders? 398
9. What is the recommended workup for diarrhea in AIDS? 398
10. Describe the clinical features of HSV proctitis in AIDS 398
11. What is the preferred endoscopic procedure for the evaluation of diarrhea in AIDS? 399
12. What is the most common cause of viral diarrhea in AIDS? 399
13. What are the treatment options for CMV enterocolitis? 400
14. Name the parasites that cause diarrhea in AIDS 400
15. Compare the clinical features and therapies for cryptosporidiosis and microsporidiosis 400
16. Which bacteria most commonly cause diarrhea in AIDS? 400
17. What is bacillary peliosis hepatis (BPH)? 401
18. Describe the management of HIV wasting syndrome 401
19. When do you initiate hepatitis B virus (HBV) therapy in the setting of HIV? 401
20. Why is it important to know the HBV treatments that are also active in treating HIV? 401
Bibliography 401
Chapter 58: Ischemic Bowel Disease 403
1. What is ischemic bowel disease? 403
2. Describe the gross anatomy of the mesenteric vascular system 403
3. An extensive collateral circulatory system exists between the systemic and splanchnic vascular networks. Describe this System 404
4. What is meant by autoregulation? 404
5. What are the different varieties of ischemic bowel disease? 405
6. What clinical circumstances predispose to ischemic bowel disease? 405
7. Describe the pathophysiology of occlusive AMI 405
8. What is abdominal angina? What is its clinical significance? 406
9. Describe the pathophysiology of nonocclusive mesenteric ischemia 406
10. What should I know about mesenteric venous occlusion as a cause of ischemic bowel disease? 406
11. What is focal segmental (short segment) ischemia? 406
12. What are the common symptoms of occlusive mesenteric ischemia? 406
13. What are the physical findings in a patient with mesenteric ischemia? 406
14. Do laboratory findings help at all? 407
15. What are the differential diagnostic considerations in a patient with suspected AMI, and how do plain abdominal radiographs help elucidate the disorder? 407
16. What is the role of magnetic resonance angiography (MRA) in patients with suspected abdominal angina? 407
17. Describe the role of Doppler ultrasound studies in diagnosis 408
18. What is the diagnostic role of endoscopy (sigmoidoscopy, colonoscopy, enteroscopy) and laparoscopy? 408
19. When should you undertake invasive mesenteric angiographic studies? 408
20. Is there any medical treatment for mesenteric ischemia? 408
21. What is the role of angioplasty and stenting in the management of ischemic bowel disease? 408
22. When should a patient with ischemic bowel disease be sent to the operating room? 408
23. What is meant by a second-look operation? 409
24. Can ischemia be isolated to the colon? 409
25. How does ischemic colitis present clinically? 409
26. How do you confirm a suspected diagnosis of ischemic colitis? 409
27. What are the sequelae of ischemic colitis? Can anything be done to modify the course of the disease? 410
28. When is surgery indicated in patients with ischemic colitis? 410
Bibliography 410
Chapter 59: Nutrition, Malnutrition, and Probiotics 411
1. What is meant by nutritional status? 411
2. Define malnutrition 411
3. How do different types of malnutrition affect function and outcome? 411
4. How do you perform a simple nutritional assessment? 411
5. Serum proteins are a marker of overall nutritional health. Which plasma proteins will have the most sensitive turnover rate? 411
6. What simple blood tests offer an instant nutritional assessment? 411
7. List desirable weights for men and women (according to the 1983 Metropolitan Height and Weight Tables) 412
8. Describe the types of commonly prescribed oral diets 413
9. What is a hidden source of calories in the intensive care unit? 413
10. Summarize the typical findings in deficiency or excess of various micronutrients 413
11. What are the nutritional concerns in patients with short bowel syndrome? 413
12. Describe the management of nutritional problems in patients with short bowel syndrome 413
13. Describe the approach to nutritional support in patients with acute pancreatitis 413
14. What adverse GI effects may be encountered in a patient using herbal supplements? 415
15. How is obesity defined, and how common is it among U.S. residents? 415
16. In 2007, what U.S. state had an obesity rate of less than 20%? 415
17. Does obesity carry a significant risk for death? 416
18. What are the medical therapies for obesity? 416
19. What are the surgical options for obesity? 416
20. What are the National Institutes of Health consensus criteria thought to be viable indications for bariatric surgery? 416
21. What is the operative mortality of GBP surgery? 416
22. What are medical benefits of bariatric surgery? 417
23. What nutritional deficiencies are seen with bariatric surgery? 417
24. Is the number of bacteria populating the human intestine greater than the total number of cells in the human body? 417
25. What value is gut microbiotica to human existence? 417
26. Is there a link between gut microbiotica and obesity? 417
27. What is the definition of a probiotic? 417
28. What are some of the common probiotics? 417
29. Have probiotics been shown to benefit the treatment of gastrointestinal disorders? 418
30. How are probiotics believed to exert beneficial effect on the gut? 418
Websites 418
Bibliography 418
Chapter 60: Pathology of the Lower Gastrointestinal Tract 420
Small Intestine 420
1. What are the morphologic features of celiac disease? 420
2. What is the differential diagnosis of the biopsy showing villous blunting? 420
3. What are the complications of celiac sprue? 421
4. Histologically, what findings suggest peptic duodentitis versus Crohn’s disease? 421
5. Discuss a few causes of infectious enteritis. 421
6. Discuss the neuroendocrine tumors. 423
Large Intestine 424
7. What are the histologic features of idiopathic inflammatory bowel disease (IBD)? 424
8. Discuss colitis-associated dysplasia in IBD. 425
9. What is the differential diagnosis of focal active colitis? 425
10. What is the differential diagnosis of pseudomembranes? 425
11. Histologically, which findings helpdifferentiate infectious colitis andNSAID-associated colitis? 425
12. What are the histologic features of microscopic colitis? 426
13. What is the differential diagnosis of polypoid lesions that can mimic adenoma? 427
Polyps And Neoplasms 427
14. What are the histologic features of conventional adenomas? 427
15. What is meant by intramucosal carcinoma in an adenoma? 428
16. What is meant by the term depressed or flat adenoma? 428
17. What is the difference between hyperplastic polyp (HP), traditional serrated adenoma(TSA), and sessile serrated adenoma (SSA)? 428
18. What are the genetic abnormalities in conventional CRCs? 428
19. What genetic abnormalities point toHNPCC? 429
20. What histologic features seen in CRCs can predict MSI-H? 430
21. What is the abnormality in MSI unstable sporadic CRCs? 430
Polyposis Syndromes 430
22. Name the hamartomatous polyp syndromes. 430
23. Name the adenomatous polyp syndromes. 430
24. How are neuroendocrine tumors classified? 431
25. What are the most common primary tumor sites that can show colon metastases? 431
26. What is the differential diagnosis of stromal tumors in colon? 431
Diseases Of The Appendix 432
27. What is the effect of IBD on the appendix? 432
28. Describe the mucinous lesions of appendix. 432
29. What is the incidence of carcinoid tumors in appendectomy specimens (performedfor appendicitis)? 432
30. What are the histologic types of mixed endocrine-exocrine neoplasms? 433
Diseases Of The Anal Canal 433
31. The typical findings of Hirschsprung disease include absence of ganglion cells.What other stain can help support the diagnosis, and what is the ideal site ofbiopsy? 433
32. How is anal intraepithelial neoplasia (AIN) graded and what is the risk of progressionto squamous cell carcinoma (SCC)? 433
33. What are the cells of origin and the immunohistochemical profile of Pagetdisease? 433
Websites 433
Bibliography 434
Chapter 61: Foreign Bodies and the Gastrointestinal Tract 435
1. How common are foreign bodies in the gastrointestinal (GI) tract? 435
2. Which populations are at risk for foreign-body ingestion? 435
3. Which areas of the GI tract lead to problems in the passage of foreign bodies? 435
4. What objects are commonly ingested? 435
5. Describe the typical clinical presentation of foreign-body ingestion 435
6. What is suggested by respiratory symptoms related to foreign-body ingestion? 435
7. Do ingested sharp objects perforate the intestine? 435
8. Why is it important to identify the type of foreign body ingested? 435
9. How urgent is removal of a foreign body after ingestion? 436
10. Describe the signs and symptoms of a complication related to foreign-body ingestion 436
11. How should foreign bodies be removed? 436
12. Which anatomic/functional defects of the GI tract contribute to foreign-body obstruction? 437
Bibliography 437
Chapter 62: Functional Gastrointestinal Disorders and Irritable Bowel Syndrome 439
1. What are functional gastrointestinal (GI) disorders? 439
2. Define irritable bowel syndrome (IBS) 439
3. Discuss the epidemiology of IBS 439
4. What is the natural history of IBS? 439
5. Discuss the current pathophysiology of IBS 439
6. Discuss the interplay of IBS and psychiatric disorders 440
7. Discuss important aspects in the patient’s history to diagnosing IBS 440
8. Discuss clinical assessment of psychological disorders 440
9. List the differential diagnosis for IBS 441
10. Discuss postinfectious IBS and related pathology 441
11. Discuss the physical exam pertinent for IBS 441
12. What is Carnett test? 441
13. What laboratory studies should be performed in all patients with IBS? 441
14. What are alarm symptoms that should alert the clinician to investigate further? 441
15. Should a colonoscopy be performed on all patients with IBS? 441
16. How is IBS diagnosed? 441
17. Discuss the general approach to patients with IBS 442
18. Describe the initial and general treatment of IBS 442
19. What medical therapies are helpful for diarrhea-predominant IBS? 442
20. What is the current role for alosetron in IBS? 442
21. What role do TCAs play in the treatment of IBS? 443
22. Discuss therapeutic options for constipation-predominant IBS 443
23. Discuss the controversy behind fiber and IBS 443
24. What is the current role for tegaserod in IBS? 443
25. What is the current role for lubiprostone in IBS? 443
26. What role do SSRIs and serotonin-norepinephrine reuptake inhibitors (SNRIs) have in the treatment of IBS? 444
27. Discuss the role of CBT and hypnotherapy in treating IBS 444
28. Do probiotics and antibiotics have a role in the management of IBS? 444
Websites 444
Bibliography 445
Chapter 63: Endoscopic Cancer Screening and Surveillance 446
1. What is endoscopic cancer screening and surveillance? 446
2. Why is endoscopic cancer screening and surveillance performed for gastrointestinal (GI) cancers? 446
Esophagus 446
3. Endoscopic cancer screening of the esophagus is primarily undertaken for what twotypes of esophageal cancer? What risk factors are associated with these two typesof cancer? 446
4. What is Barrett (metaplasia) esophagus? Why is endoscopic screening andsurveillance for Barrett’s esophagus necessary? 446
5. Which patients should undergo endoscopic screening for Barrett’s esophagus? 446
6. What techniques are used to perform endoscopic screening in Barrett’sesophagus? 446
7. What is the rationale for endoscopic surveillance in Barrett’s esophagus? 446
8. What techniques are used to perform endoscopic surveillance in Barrett’s esophagus? 447
9. How often should patients with Barrett’s esophagus undergo endoscopic surveillance? 447
10. How do you manage low-grade dysplasia (LGD) in patients with Barrett’s esophagus? 447
11. How do you manage HGD in patients with Barrett’s esophagus? 447
12. What is the principal role of endoscopic ultrasound (EUS) in evaluating patients with HGD? 447
13. What is the next step if adenocarcinoma is identified while performing endoscopicsurveillance for Barrett’s esophagus? 447
14. What new imaging modalities are available for Barrett’s esophagus endoscopicscreening and surveillance? 448
15. Do patients with achalasia have an increased risk of esophageal cancer? 448
16. What is the role of endoscopic cancer surveillance in patients with achalasia? 448
17. Is there a link between caustic ingestion and the development of esophagealcancer? 448
18. What are the clinical characteristics of patients who develop esophageal cancerafter a caustic injury? 448
19. How is endoscopic surveillance used in patients with a history of caustic ingestion? 448
20. What rare genetic disorder is associated with a high incidence of SCC of the esophagus? 448
21. What type of endoscopic surveillance is recommended in patients with tylosis? 448
22. Are patients with a history of head and neck, lung, or esophageal SCC at risk forsynchronous or metachronous cancer of the esophagus? 448
23. Are endoscopic screening and surveillance warranted in patients with aerodigestiveSCC? 448
Stomach And Small Bowel 449
24. What is the malignant potential of gastric polyps? 449
25. How are gastric polyps managed when encountered radiographically or endoscopically? 449
26. Is endoscopic surveillance required after the removal of a gastric polyp? 449
27. What is gastric intestinal metaplasia (GIM)? 449
28. How common is GIM? What is its malignant potential? 449
29. What role does endoscopic surveillance have in GIM? 449
30. Are patients with pernicious anemia at an increased risk for gastric cancer? Isendoscopic screening or surveillance required? 449
31. What is the frequency of gastric cancer in patients who have undergone a partialgastrectomy? 449
32. What are the endoscopic surveillance recommendations for postgastrectomy surgerypatients? 449
33. Who is at risk for ampullary and nonampullary duodenal adenomas? 450
34. What is the upper GI tract endoscopic surveillance strategy for patients with FAP? 450
35. How often is surveillance endoscopy performed on patients who have undergoneendoscopic resection of ampullary adenomas? 450
36. When should surveillance endoscopy begin for patients with PJS? 450
37. What is the role of capsule endoscopy in small bowel surveillance for PJS? 450
38. What are the endoscopic surveillance guidelines for sporadic duodenal adenomas? 450
Pancreas 450
39. Who should undergo endoscopic screening and surveillance for pancreatic cancer? 450
40. When should endoscopic screening begin for patients at increased risk forpancreatic neoplasia? 450
41. What is the recommended endoscopic surveillance interval for patients at high riskfor pancreatic cancer? 451
Colon 451
42. At what age is CRC screening recommended for average-risk patients? What are thepreferred testing modalities for CRC screening? 451
43. When should endoscopic screening begin for individuals with a family history ofCRC? How often should endoscopic surveillance be performed in these individuals? 451
44. What are the endoscopic surveillance guidelines for individuals with a personalhistory of colon cancer? 451
45. Outline the endoscopic surveillance guidelines for individuals with a personal historyof rectal cancer 451
46. What is the role of EUS in the endoscopic surveillance of individuals with a personalhistory of rectal cancer? 451
47. Do individuals with a first-degree relative diagnosed with adenomatous polypsrequire earlier screening for CRC? Do they have an increased risk for CRC? 451
48. What are the surveillance recommendations for a patient with a previous history ofadenomatous colon polyps? 451
49. Define familial adenomatous polyposis syndrome. What is the risk of developing CRCin patients with FAP syndrome? 452
50. When should endoscopic screening begin in patients with FAP? 452
51. Do patients with PJS require endoscopic screening and surveillance for CRC? 452
52. What is hereditary nonpolyposis colorectal cancer syndrome (HNPCC)? 452
53. What are the endoscopic screening and surveillance guidelines for HNPCC? 452
54. Do patients with ulcerative colitis (US) and Crohn’s disease require endoscopicsurveillance? 452
55. Which clinical characteristics increase the risk of CRC in patients with UC andCrohn’s disease? 452
56. How should endoscopic surveillance be performed in patients with UC and Crohn’sdisease? 452
57. What is the treatment strategy for dysplasia in patients with UC or Crohn’s disease? 452
58. How are adenomatous-appearing polyps managed in patients with UC and Crohn’sdisease? 453
Websites 453
Bibliography 453
Chapter 64: Rheumatologic Manifestations of Gastrointestinal Diseases 455
Enteropathic Arthritis 455
1. How often does an inflammatory peripheral or spinal arthritis occur in patients withidiopathic inflammatory bowel disease (IBD)? 455
2. What are the most common joints involved in ulcerative colitis and Crohn’s diseasepatients with an inflammatory peripheral arthritis? 455
3. Describe the clinical characteristics of the inflammatory peripheral arthritisassociated with idiopathic IBD. 455
4. What other extraintestinal manifestations commonly occur in patients with idiopathicIBD and inflammatory peripheral arthritis? 456
5. Do the extent and activity of IBD correlate with the activity of the peripheralinflammatory arthritis? 456
6. Which points in the history and physical examination are helpful in separatinginflammatory spinal arthritis from mechanical low back pain in an IBD patient? 456
7. Does the activity of inflammatory spinal arthritis correlate with the activity of the IBD? 456
8. What human leukocyte antigen (HLA) occurs more commonly than expected inpatients with inflammatory spinal arthritis associated with IBD? 456
9. What serologic abnormalities are seen in patients with IBD? 456
10. Describe the typical radiographic features of inflammatory sacroiliitis and spondylitisin IBD patients. 457
11. What other rheumatic problems occur with increased frequency in IBD patients? 457
12. Can treatment alleviate the symptoms of inflammatory peripheral arthritis and/orspinal arthritis in IBD patients? 458
13. What rheumatic disorders are associated with pouchitis, lymphocytic colitis (LC),and/or collagenous colitis (CC)? 458
14. Why are patients with IBD more prone to develop an inflammatory arthritis? 458
Reactive Arthritis 458
15. What is reactive arthritis, and what are the most common GI pathogens that cause it? 458
16. Which joints are most commonly involved in a reactive arthritis following a bowelinfection (i.e., postenteritic reactive arthritis)? 459
17. Describe the clinical characteristics of postenteritic reactive arthritis. 459
18. What extra-articular manifestations can occur in patients with postenteriticreactive arthritis? 459
19. How commonly do patients with postenteritic reactive arthritis have the clinicalfeatures of Reiter syndrome? 459
20. How do the radiographic features of inflammatory sacroiliitis and spondylitis due topostenteritic reactive arthritis differ from those in IBD patients? 459
21. Discuss the relationship of HLA-B27 positivity in patients with postenteritic reactivearthritis compared with a normal healthy population. 460
22. Explain the current theory for the pathogenesis of a postenteritic reactive arthritis. 460
23. Is any therapy beneficial for postenteritic reactive arthritis? 460
Whipple Disease 461
24. Who was Whipple? 461
25. What are the multisystem manifestations of Whipple disease? 461
26. Describe the clinical characteristics of the arthritis associated with Whippledisease. 461
27. What is the etiology of Whipple disease? 461
28. How is Whipple disease best treated? 461
Other Gastrointestinal Diseases 462
29. What rheumatic manifestations have been described in patients with celiac disease(gluten-sensitive enteropathy)? 462
30. Describe the intestinal bypass arthritis-dermatitis syndrome. 462
31. What types of arthritis can be associated with carcinomas of the esophagus and colon? 462
32. What are the clinical features of the pancreatic panniculitis syndrome? 462
33. What musculoskeletal problem can occur with pancreatic insufficiency? 462
Bibliography 462
Chapter 65: Dermatologic Manifestations of Gastrointestinal Disease 464
1. At what serum level of bilirubin do adults and infants develop clinically noticeable jaundice? 464
2. Where is clinical jaundice first visible? 464
3. What other conditions produce yellowish discoloration of the skin? 464
4. What are Terry nails and Muehrcke nails? 464
5. What gastrointestinal disease is associated with blue lunulae? 464
6. What are spider angiomas? Why are they associated with liver disease? 464
7. Do the number of spider angiomas correlate with the severity of alcohol-induced liver disease? 465
8. Why do many patients with hepatobiliary disease itch? 465
9. A 64-year-old alcoholic man presents with blisters on the dorsal hands and sclerotic changes of the facial skin. For what chronic liver disease should he screened? 465
10. A 25-year-old woman presents with painful, tender, red-to-violaceous subcutaneous nodules of the pretibial skin associated with diarrhea. What is the skin lesion? 465
11. A 22-year-old woman presents with low-grade fever and an expanding oozing ulcer of the hand that is rapidly increasing in size despite aggressive surgical debridement and intravenous antibiotics. What does this patient have? 465
12. List the GI diseases most commonly associated with pyoderma gangrenosum 466
13. What are the cutaneous manifestations of pancreatitis? 466
14. A 32-year-old man presents with a 2-year history of recurrent blisters that are intensely pruritic and have been recalcitrant to antihistamines and topical corticosteroids. They are primarily located on the elbows, knees, and buttocks. What does this pa 466
15. What GI disease is most commonly associated with dermatitis herpetiformis? 466
16. A 30-year-old man presents with acute GI bleeding. He has yellowish pebbly papules that coalesce into plaques of the neck, antecubital fossae, and axillae. Similar lesions are also present on his lower lip. What does he have? 467
17. A 24-year-old man presents with a history of unexplained melena, nose bleeds, and red macular lesions of his lips and fingers. What does he have? 467
18. During evaluation for GI bleeding, a 25-year-old man is noted to have 2- to 4-mm pigmented macules of the lips and buccal mucosa. What does he most likely have? 467
19. During evaluation for numerous polyps of the colon, a 19-year-old man is noted to have multiple cysts of the skin and an osteoma. What does he most likely have? 467
20. A 44-year-old man presents with multiple hamartomatous polyps of the small and large bowel. Cutaneous examination reveals cobblestoning of the oral mucosa and multiple small papules and verrucous papules of this face. What does this patient most likely 467
21. A 60-year-old man has had multiple keratoacanthomas removed from his skin and recently had a biopsy of a sebaceous adenoma of the cheek. For what syndrome should he be evaluated? 468
22. What is Trousseau sign? 468
23. A 50-year-old woman presents with alopecia, unexplained 20-pound weight loss, and very superficial flaccid vesicles and erosions on an erythematous base that preferentially involves the perioral and perianal areas. What does she most likely have? 468
24. Who was Sister Mary Joseph and what is a Sister Mary Joseph nodule? 468
Website 468
Bibliography 469
Chapter 66: Endocrine Aspects of Gastroenterology 470
1. What are the etiologies of diabetic gastroparesis? How should it be treated? 470
2. Describe the mechanisms of chronic diarrhea in diabetes mellitus and their treatments 470
3. Patients with primary biliary cirrhosis (PBC) are at increased risk for what endocrine disorders? 470
4. What are the most prominent GI manifestations seen in hyperthyroidism and in hypothyroidism? 471
5. Name the two metabolic causes of acute pancreatitis. Which diabetic medication may cause acute pancreatitis? 471
6. Define hypoglycemia and list the counter-regulatory response to hypoglycemia 471
7. What is Whipple triad? Why is it important? 472
8. Which GI and hepatic disturbances are associated with hypoglycemia? 472
9. How does ethanol cause hypoglycemia? 472
10. Should routine screening for polyps and colorectal cancer be performed in patients with acromegaly? 472
11. What GI symptoms may be seen with hypercalcemia? 473
12. What are the National Institutes of Health (NIH) criteria for bariatric surgery? 473
13. How effective are bariatric surgical procedures for long-term control of morbid obesity? 473
14. Describe gut hormone changes following Roux-en-Y gastric bypass (RYGB) that theoretically may explain the developmen that theoretically may explain the development of nesidioblastosis 473
15. What is multiple endocrine neoplasia type 1 (MEN1)? 474
16. List the order of pancreatic islet cell tumor prevalence in MEN1 and their clinical manifestations 474
17. Are patients with MEN1 and gastrinomas at greater risk for esophageal complications compared with gastrinoma patient without MEN1? 474
18. Define carcinoid syndrome 474
19. How are carcinoid tumors classified? 474
20. Where do carcinoid tumors occur, and what are the characteristic features of carcinoid tumors by site of origin? 474
21. How is carcinoid syndrome treated? 475
22. What hepatic effects are seen in patients with adrenal disorders? 475
Bibliography 475
Chapter 67: Radiography And Radiographic-Fluoroscopic Contrast Examinations 477
1. When requesting an imaging examination, what information should a clinician provide for a radiologist? 477
Abdominal Radiography 477
2. Which radiographs should constitute an acute abdominal series? 477
3. What is the key radiographic finding of bowelobstruction? 478
4. What are causes of pneumatosis intestinalis? 478
5. What distinguishes portal venous gas from pneumobilia? 478
Contrast Media 479
6. When is barium preferable to iodinated contrast media to opacify the lumen of thegastrointestinal (GI) tract? 479
7. What is the role of iodinated (water-soluble) contrast for opacification of the lumenof the GI tract? 480
8. Are some iodinated contrast media better than others? 480
Swallowing Studies 481
9. What is a barium swallow? 481
10. What can a barium swallow contribute to an evaluation for dysphagia? 481
11. Which esophageal motility disorders are diagnosable by barium swallow? 482
12. What may a barium swallow contribute to diagnosis and managementof GERD? 482
13. How can a barium swallowdistinguish achalasia fromscleroderma? 482
14. What findings help distinguishachalasia secondary to cancerfrom primary achalasia? 482
Upper Gastrointestinal Series 483
15. Can benign and malignant gastric ulcers be distinguished? 483
Small Bowel 483
16. What are advantages and disadvantages of, and indications for, enteroclysis (smallbowel enema)? 483
17. When information from imaging, beyond that provided by radiographs, is indicatedfor suspected small bowel obstruction, which fluoroscopic-radiographic contrastexamination is best? 484
18. When is CT preferable to a fluoroscopic-radiographic contrast study for small bowelobstruction? 484
19. When is a retrograde examination of small bowel indicated? 485
Colon And Rectum 485
20. What are indications for single-contrast and double-contrast techniques of bariumenema examination? 485
21. What are advantages and disadvantages of screening for colon cancer with a bariumenema instead of colonoscopy? 485
22. What is the role of defecography (evacuation proctography)? 485
Cholangiopancreatography 485
23. What cholangiopancreatographic features distinguish pancreatitis from ductaladenocarcinoma of the pancreatic head? 485
24. What is the double duct sign of cholangiopancreatography? 485
25. What pancreatographic features distinguish pancreas divisum from completeobstruction of the main pancreatic duct? 487
Other 487
26. What are advantages of fistulography? 487
Websites 487
Bibliography 488
Chapter 68: Interventional Radiology: Cross-Sectional Imaging Procedures 489
1. What common percutaneous procedures are performed using cross-sectional imaging guidance? 489
2. What materials and equipment are used for FNAs, core biopsies, and percutaneous catheter drainages? 489
3. What are the indications for percutaneous image-guided biopsy? 489
4. What four conditions must be satisfied before a percutaneous procedure can be performed? 489
5. What coagulation parameters are assessed before a percutaneous procedure? 489
6. Which imaging modalities are used to guide interventional procedures? 490
7. Summarize the advantages and disadvantages of US 490
8. Summarize the advantages and disadvantages of CT 491
9. Summarize the advantages and disadvantages of conventional fluoroscopy 491
10. Summarize the advantages and disadvantages of MRI 492
11. What two techniques can be used to drain fluid collections? 492
12. Which technique is used more often? 492
13. What pharmacologic agents can be injected into septated or viscous abdominal fluid collections to improve drainage? 492
14. What should you suspect if the drainage catheter has persistently elevated outputs? 492
15. When should you remove the drainage catheter? 492
16. What are the major complications of percutaneous procedures? 492
17. How common is seeding of the needle tract during routine tumor biopsy? 493
Hepatic Interventions 493
18. What image-guided procedures are performed in the liver? 493
19. How is hepatic metastatic disease diagnosed? 493
20. How is malignant, primary hepatic neoplasm diagnosed? 493
21. How are pyogenic hepatic or parahepatic abscesses treated? 493
22. Describe the treatment of simple, benign, epithelialized hepatic cysts 493
23. Is FNA or core biopsy safe for all hepaticlesions? 494
24. Describe the percutaneous thermal ablative treatments for HCC? 495
25. What are the advantages of RFA and other methods of percutaneous thermalablation? 496
26. What are the contraindications of RFA or percutaneous thermal ablative techniques? 496
27. Describe the association of Childs-Pugh score and survival in patients with HCCtreated with RFA. 496
28. Describe the risks of RFA related to the anatomic location of the tumor? 496
29. In the treatment of HCC, how do survival outcomes of RFA compare to surgicalresection? 496
30. Describe the advantages of combining RFA with transcatheter arterialchemoembolization (TACE) in the treatment of HCC? 496
31. What other liver cancers have been treated with percutaneous thermal ablativetechniques? 496
Splenic Interventions 497
32. What interventions are possible in the spleen? 497
Pancreatic Procedures 497
33. What procedures are appropriate for solid pancreatic masses? 497
34. What procedures are used for pancreatic fluid collections? 498
35. What precautions apply to percutaneous drainage of pancreatic fluid collections? 498
Adrenal Biopsy 499
36. What is the role of adrenal gland biopsy? 499
Bibliography 499
Chapter 69: Interventional Radiology: Fluoroscopic and Angiographic Procedures 500
Hepatic Transarterial Chemoembolization 500
1. Define hepatic transarterial chemoembolization. 500
2. How safe is hepatic TACE? 500
3. Why would TACE be used to treat patients with hepatic malignancy? 500
4. How effective is TACE? 501
Yttrium-90 (90Y)-Radioembolization 501
5. Describe 90Y-radioembolization. 501
6. Name the two FDA-approved and commercially available radioactive microspheresand describe their differences. 501
7. Compare and contrast radioembolization and TACE. 501
8. What are the contraindications of radioembolization? 502
Biliary Procedures 502
9. Is percutaneous transhepatic biliary drainage the primary method to treat biliaryobstruction? 502
10. What are the indications for PTBD? 502
11. What particular problems are involved in the treatment of hilar obstruction? 502
12. Why is endoscopic drainage difficult in patients with biliary obstruction afterbiliary-enteric anastomosis? 502
13. Describe the approach to bile duct injuries due to laparoscopic cholecystectomy. 502
14. Explain the advantages and disadvantages of using metallic stents for the treatmentof biliary obstruction. 502
15. What are the indications for percutaneous cholecystostomy? 503
Gastrointestinal Bleeding 503
16. When do diagnostic angiography and percutaneous transcatheter therapy play a rolein the management of gastrointestinal (GI) bleeding? 503
17. How important is localization of the bleeding site before angiography? 503
18. What two types of transcatheter therapy are used for GI bleeding? 504
19. What agents are used for transcatheter embolization? 504
Transjugular Liver Biopsy 504
20. What are the specific indications for transjugular liver biopsy? 504
21. How is it performed? 504
22. Why is it important to biopsy via the right hepatic vein and not the middlehepatic vein? 504
Transjugular Intrahepatic Portosystemic Shunt 505
23. What is TIPS? How is it performed? 505
24. What are the benefits of successful TIPS? 505
25. What are the indications for TIPS? 505
26. What are the contraindications to performing the TIPS procedure? 505
27. What is the technical success rate for TIPS? What are the most common causes of afailed procedure? 505
28. How effective is the TIPS procedure for controlling variceal hemorrhage? 505
29. What are the morbidity and mortality rates for TIPS? 506
30. Describe the major long-term complication of TIPS. How is it treated? 506
31. How is shunt patency followed? 506
Bibliography 507
Chapter 70: Nonivasive Gastrointestinal Imaging: Ultrasound, Computed Tomography, Magnetic Resonance Imagine 508
Liver Imaging 508
1. How is segmental liver anatomy defined? 508
2. How has the advent of multidetector computed tomography (MDCT) changed theevaluation of the liver, pancreas and biliary system? 508
3. What is CT arterial portography? 508
4. What causes fatty filtration of the liver? 508
5. Describe the imaging findings of fatty infiltration of the liver. 509
6. Describe the imaging findings in cirrhosis. 509
7. Define primary and secondary hemochromatosis. 510
8. Which is the most sensitive exam in detecting hemochromatosis? 510
9. How do liver metastases appear on different imaging modalities? 510
10. What MRI contrast agents are available for use in hepatobiliary imaging? 510
11. What are the three growth patterns of HCC? 512
12. How does HCC appear on different imaging modalities? 512
13. What is the most common benign neoplasm of the liver? 513
14. Describe the imaging characteristics of hepatic hemangiomas. 513
15. Outline the workup for a suspected cavernous hemangioma. 513
16. How can FNH and hepatocellular adenoma (HCA) be differentiated? 514
17. Describe the appearance of FNH on imaging modalities. 514
18. How does HCA appear on imaging modalities? 515
19. Describe the appearance of a hepatic abscess on imaging. 515
Doppler Imaging Of The Liver 515
20. What is a normal Doppler waveform? 515
21. Describe the sonographic findings of portal hypertension on Dopplerwaveforms. 515
22. How are Doppler waveforms altered inportal vein thrombosis? 516
23. How does Budd-Chiari syndrome affect Doppler waveforms? 516
24. Discuss the role of US in the evaluation of transjugular intrahepatic portosystemicshunts (TIPS). 516
Biliary Tract Imaging 516
25. Describe the sonographic findings in acute cholecystitis 516
26. What other conditions can result in gallbladder wall thickening? 516
27. Describe the radiologic workup of suspected biliary tree obstruction. 517
28. What is MRCP? What advantages does it have compared with ERCP? 517
29. Describe the differential imaging features seen in the common causes of biliaryobstruction. 518
Pancreatic Imaging 518
30. How can acute pancreatitis be distinguished from chronic pancreatitis on imaging? 518
31. Describe the role of CT and US in assessing the complications of pancreatitis. 519
32. What are the imaging findings of pancreatic ductal adenocarcinoma? 520
33. Which imaging modality is best for detecting and staging pancreatic cancer? 520
34. What are the CT criteria for unresectability of pancreatic carcinoma? 520
35. What are the characteristic features of the major cystic pancreatic neoplasms? 520
Abdominal And Pelvic Imaging 521
36. How is simple ascites distinguished from complicated ascites? 521
37. How do you differentiate abdominal fluid from pleural fluid? 521
38. How has MDCT changed evaluation of the small bowel? 521
39. How is CT used to evaluate the large bowel? 523
40. Describe the optimal radiographic workup of diverticulitis. 523
41. What are the CT and US findings of acuteappendicitis? 524
42. Which examination is better for diagnosing acute appendicitis? 524
43. Discuss the role of imaging in the assessment of intra-abdominal abscess 524
44. What is CT or virtual colonoscopy and how effective is it in screening for polyps? 524
Aids-Related Disorders 524
45. What characteristic features of AIDS are seen in the biliary system? 524
46. Describe the imaging features of AIDS in the liver. 525
47. What extrahepatic manifestations of AIDS in the GI tract can be noted by imaging? 525
Bibliography 525
Chapter 71: Nuclear Imaging 527
1. Outline the general advantages of nuclear medicine procedures compared with other imaging modalities. 527
2. What are the disadvantages of nuclear medicine procedures compared with other radiographic studies? 527
3. What nuclear medicine tests are most helpful in gastrointestinal (GI) medicine? 527
4. How is cholescintigraphy (hepatobiliary imaging) performed? What is a normal study? 527
5. How should patients with acute cholecystitis be prepared? What manipulations are used to shorten the study or increase its reliability? 527
6. How is cholescintigraphy used to diagnose and manage biliary leak? 528
7. What is the role of cholescintigraphy in diagnosing biliary atresia? 529
8. What is gallbladder dyskinesia? How does cholescintigraphy evaluate the emptying of the gallbladder? 530
9. What nuclear medicine esophageal studies are available? How are they used? 530
10. What is a nuclear medicine gastric emptying study? 531
11. What is the role for nuclear medicine studies in evaluating hepatic mass lesions? 531
12. How can nuclear medicine procedures assist in detecting ectopic gastric tissue? 532
13. Can accessory splenic tissue or splenosis be detected via nuclear medicine procedures? 532
14. Which nuclear medicine procedures are useful in localizing lower GI bleeding? 533
15. Are nuclear medicine procedures clinically useful in localizing GI bleeding, or are simpler techniques adequate? 533
16. Is nuclear medicine helpful in placement of arterial perfusion catheters? 534
17. Are there additional minimally invasive treatments for unresectable malignant liver masses? 534
18. Can abdominal malignancies be evaluated with nuclear medicine studies? 534
19. What is PET and how does it work? 534
20. What malignancies can PET and PET/CT be used for? 535
Bibliography 536
Chapter 72: Endoscopic Ultrasound 537
1. When was intraluminal gastrointestinal (GI) ultrasound (US) first performed? 537
2. How do US waves visualize the GI tract? 537
3. How does the frequency of the US beam influence the depth of beam penetration and image resolution? 537
4. What are the ultrasonographic properties of the common structures of the body? 537
Normal Anatomy 537
5. What determines the thickness of the echosonographic layer visualized? What is thenormal endosonographic anatomy of the intestinal wall? 537
6. What are the imaging characteristics of normal and malignant lymph nodes on EUS? 538
7. How are blood vessels distinguished from lymph nodes on EUS? 538
8. Describe the normal EUS anatomy of the retroperitoneum. What are its major landmarks? 538
9. What are the indications for EUS examination? 539
10. How is EUS used in the clinical evaluation of esophageal cancer? 539
11. How can EUS findings affect clinical management of esophageal carcinoma? 539
12. What are the problematic areas for EUS in the staging of esophageal cancer? 539
13. Does EUS have a role in the evaluation of gastric cancer? 540
14. What are the problematic areas for EUS staging of gastric malignancy? 540
15. Summarize the TNM staging classification for gastric malignancy. 540
16. How does staging affect treatment? 541
17. Is EUS helpful in the evaluation of gastric lymphoma? 541
18. How is EUS helpful in evaluating pancreatic neoplasms? 541
19. Neuroendocrine tumors (NETs) of the pancreas and peripancreas are often difficult tolocalize by conventional CT, US, and angiography. Does EUS examination offer anyvalue in localizing these tumors? 541
20. Describe the use of EUS in the evaluation of colon malignancy. 541
21. Describe the use of EUS in the evaluation of rectal malignancy. 541
22. Summarize the EUS characteristics of submucosal tumors (SMTs). 542
23. Is EUS useful in the evaluation of nonneoplastic disease? 542
24. How is EUS used in the evaluationof patients with portalhypertension? 543
25. Does EUS have a role in the evaluation of recurrent idiopathic pancreatitis? 543
26. What is the stack sign? Does it have clinical significance? 543
27. Describe the role of EUS in the evaluation of chronic pancreatitis. 543
28. Summarize the EUS criteria for chronic pancreatitis. 544
29. What is the role of EUS in the evaluation of autoimmune pancreatitis (AIP)? 544
30. Discuss the role of EUS in evaluating patients with common bile duct stones. 544
Websites 544
Bibliography 544
Chapter 73: Advanced Therapeutic Endoscopy 545
1. What is advanced therapeutic endoscopy? 545
2. What are the major advanced therapeutic endoscopy techniques? 545
3. What are the applications of EMR and ESD? 546
4. How is EMR performed? 547
5. How is ESD performed? 547
6. What are the differences/limitations of the EMR and ESD? 547
7. What are the complications of EMR/ESD? 547
8. What are some investigational applications of EMR/ESD? 548
9. What are some investigational applications of advanced endoscopic ultrasound? 548
10. What is the role of EUS-guided fine-needle aspiration (FNA) biopsy in tissue sampling? Sampling of nodes? 548
11. How is EUS-FNA performed? 548
12. What are the advantages of EUS-FNA over other sampling modalities? 548
13. What are the sensitivity and specificity of EUS-FNA for the diagnosis of malignancy? 549
14. What is the role of EUS-FNA in the evaluation of mediastinal lymphadenopathy? 549
15. What are the risks of EUS-FNA? 549
16. What is the role of EUS in sampling pancreatic cystic neoplasms? 549
17. Is there a risk of biopsy tract seeding when EUS-FNA of a suspected malignancy is sampled? 549
18. How is EUS-guided transmural pseudocyst drainage performed? 549
19. What are the indications for EUS-guided celiac plexus block (CPB) and celiac plexus neurolysis (CPN)? What is the difference? Why do they work? 550
20. How are EUS-guided celiac plexus block and neurolysis performed? 551
21. What is the success rate of CPN? CPB? 551
22. What are the potential complications of CPN? 551
23. Is EUS-guided cholangiography or pancreatography possible? When are they indicated? 552
24. What is high-frequency US-probe sonography-assisted EMR? 552
Websites 552
Bibliography 552
Chapter 74: Surgery: Gastroesophageal Reflux And Esophageal Hernias 553
Gastroesophageal Reflux Disease 553
1. Define gastroesophageal reflux disease (GERD). 553
2. Describe the typical and atypical symptoms of GERD. 553
3. What factors play a role in altering the gastroesophageal (GE) barrier? 553
4. Describe the workup of patients with suspected GERD. 553
5. What is the significance of a defective LES? 554
6. What is the significance of abnormal esophageal motility in patients with GERD? 554
7. What is Barrett’s esophagus and what are the risk factors? 554
8. What are the indications for an antireflux operation? 554
9. What are the surgical options to relieve GERD? 554
10. What are the important technical steps of a Nissen fundoplication? 555
11. What are the predictors of successful antireflux surgery? 555
12. What are the predictors of poor outcome after antireflux surgery? 555
13. Explain the benefits of surgical treatment of GERD. 555
14. What are the complications of laparoscopic fundoplication? 556
Paraesophageal Hernias 556
15. Define the four types of hernias occurring at the hiatus. 556
16. What causes a hiatal hernia? 557
17. What are the signs and symptoms of a paraesophageal hernia? 557
18. How are hiatal and paraesophageal hernias diagnosed and evaluated? 557
19. What are the indications for surgical repair of paraesophageal hernias? 557
20. What is the operative strategy of a paraesophageal hernia repair? 557
Website 557
Bibliography 558
Chapter 75: Surgery: Achalasia and Esophageal Cancer 559
Achalasia 559
1. Define achalasia. What are the classic findings of esophageal achalasia? 559
2. What are the most common symptoms of achalasia? 559
3. What is pseudoachalasia? How is it diagnosed? 559
4. What is vigorous achalasia? 559
5. What are the nonsurgical options for treatment of achalasia? 559
6. What are the basic components of laparoscopic Heller myotomy for achalasia? 559
7. How do long-term results of Heller myotomy compare with mechanical esophagealdilatation? 560
8. Describe the complications of Heller myotomy. 560
9. Summarize the treatment algorithm for patients with achalasia. 560
10. What is the association between achalasia and esophageal cancer? 560
Esophageal Cancer 560
11. What is the incidence of esophageal cancer? 560
12. What are the risk factors of esophageal cancer? 560
13. Describe the relationship of Barrett’s esophagus to esophageal cancer. 560
14. Can Barrett’s esophagus regress after antireflux therapy? 561
15. Discuss the surgical management of patients with high-grade dysplasia. 561
16. What are the surgical approaches to the patient with esophageal cancer? 561
17. When is neoadjuvant therapy appropriate in the treatment of patients withesophageal carcinoma? 561
18. Describe nonsurgical options for treatment of esophageal cancer. 561
19. What is the survival of patients with esophageal cancer? 562
Bibliography 562
Chapter 76: Surgery For Peptic Ulcer Disease 563
1. Describe the five types of gastric ulcer in terms of location, gastric acid secretory status, incidence, and complications 563
2. Describe the classic indications and goals for peptic ulcer surgery 563
3. What are the three classic operations used for PUD? 563
4. Describe the truncal vagotomy, selective vagotomy, and highly selective vagotomy 563
5. Why is an outlet or drainage procedure added to truncal vagotomy? What are the surgical options? 564
6. What are the relative indications and contraindications to highly selective vagotomy? 565
7. What are the surgical options for reconstruction after antrectomy? 565
8. How is the type of reconstruction determined for a given patient? 565
9. Define intractability in terms of the medical treatment of PUD? 565
10. Describe the most appropriate elective operative procedure for duodenal ulcers and each type of gastric ulcer 566
11. Describe the presentation of a patient with a perforated peptic ulcer 566
12. Why do almost all perforated gastric ulcers require an operation? 566
13. What are the contraindications to medical management of perforated PUD? 566
14. What are the three major goals of operation for perforated PUD? 566
15. What is the preferred operation for treatment of perforated gastric ulcer? 567
16. What is the preferred operation for treatment of a perforated duodenal ulcer? 567
17. What are the major risk factors for mortality in the surgical treatment of perforated PUD? 567
18. Discuss the role for laparoscopy in the management of perforated PUD and the indications for conversion to an open operation 567
19. In patients with GI bleeding caused by PUD, what are the predictors for rebleeding in the hospital? What is the Forrest classification? 567
20. What are the classic indications for operation for rebleeding after endoscopic therapy? 567
21. What are the operative options for control of a bleeding gastric ulcer? 568
22. What is the most appropriate surgical procedure for a bleeding duodenal ulcer? 568
23. How is gastric outlet obstruction due to PUD surgically managed? 568
24. Discuss the role for endoscopic and laparoscopic management of GOO secondary to PUD 568
25. What are the long-term outcomes and risks for complications after truncal vagotomy and drainage, truncal vagotomy and antrectomy, and highly selective vagotomy? 568
26. What are the Visick criteria? 569
27. How should postoperative gastroparesis be managed? 569
28. Describe the management of duodenal stump disruption (blow-out) after truncal vagotomy, antrectomy, and Billroth II reconstruction 569
29. What is dumping syndrome? Describe the pathophysiology and treatment 569
30. Describe the pathophysiology of bile reflux gastritis. How is it managed? 569
31. What is the presentation of Zöllinger-Ellison syndrome? 570
32. How is Zöllinger-Ellison syndrome diagnosed? 570
33. For which patients with Zöllinger-Ellison syndrome is operative intervention indicated? 570
34. Describe the preoperative evaluation for gastrinoma 570
35. Where is the gastrinoma triangle? What percentage of tumors occur in this area? 571
36. Describe the operative scheme for exploration, localization, and removal of gastrinoma 571
37. Describe the risk of gastric stump cancer after partial gastrectomy for duodenal and gastric ulcer 571
Bibliography 571
Chapter 77: Surgical Approach To The Acute Abdomen 572
1. What is the significance of the term acute abdomen? 572
2. What are the critical factors in the history of present illness? 572
3. Which disorders are associated with specific age groups? 572
4. Summarize the significance of pain location 572
5. What associated problems help to pinpoint the diagnosis? 572
6. What is the peritoneum and its pain innervation, and what are peritoneal signs? 572
7. What is the significance of rebound pain, and should it be elicited? 572
8. What is the nature of intestinal pain? 573
9. How does the duration of pain help in making a diagnosis? 573
10. Is acute abdomen ruled out by absence of fever or leukocytosis? 573
11. What is the significance of bowel sounds? 573
12. What is the most important part of the abdominal examination? 573
13. What are the psoas and obturator signs? 573
14. What is Rovsing sign? 573
15. What is Kehr’s sign? 573
16. Define mittelschmerz 573
17. How does urinalysis help in the assessment? 573
18. What should be the first imaging study obtained? 573
19. How is ultrasound (US) used? 573
20. What additional imaging studies may help in the diagnosis? 573
21. If the diagnosis is in doubt, what other procedure should be done? 574
22. Is exploratory laparotomy justified, even if it produces no significant findings? 574
23. Is exploratory laparoscopy useful in the setting of acute abdomen? 574
24. In blunt trauma, CT scan of the abdomen and pelvis reveals free peritoneal fluid collections. When is observation appropriate instead of immediate surgical exploration? 574
25. Do all penetrating injuries to the abdomen require laparotomy? 574
26. What is the role of laparoscopy in trauma? 574
27. When is surgery indicated for peptic ulcer disease (PUD)? 574
28. When is cholecystectomy optimal for acute pancreatitis, presumably due to gallstone disease? 574
29. When is surgery indicated for severe acute pancreatitis? 574
30. Describe treatment for a pancreatic pseudocyst 575
31. What is the best method to diagnose pain secondary to mesenteric ischemia? 575
32. Describe the surgical strategy for the treatment of Crohn’s disease 575
33. When should surgery be offered for uncomplicated acute diverticulitis? 575
34. Should elderly patients with sigmoid or cecal volvulus undergo surgery? 575
35. How should toxic megacolon in the setting of ulcerative colitis be managed? 575
36. How should Ogilvie’s syndrome be managed? 575
37. After endoscopic retrograde cholangiopancreatography (ERCP), a patient develops upper abdominal and back pain. What steps should be considered? 575
38. How should esophageal perforation be managed after endoscopy? What if the patient has achalasia? Esophageal carcinoma? 575
39. How should colonic perforation be managed after colonoscopy? 576
Bibliography 576
Chapter 78: Colorectal Surgery: Polyposis Syndromes And Inflammatory Bowel Disease 577
1. Name four different types of intestinal polyps. 577
2. What is a hamartoma? 577
3. Which intestinal polyposis syndromes are associated with hamartomatous polyps? 577
4. How is Peutz-Jeghers syndrome manifest? 577
5. Describe the manifestation of familial adenomatous polyposis (FAP). 577
6. What is Gardner syndrome? 577
7. How does one screen for FAP? 577
8. What are the surgical indications for ulcerative colitis? 578
9. What are the elective surgical options for FAP and chronic ulcerative colitis? 578
10. Can one always tell the difference between Crohn’s disease and ulcerative colitis? 578
11. What is pouchitis? How is it treated? 578
12. Does a defunctionalized colon develop colitis? 578
13. What type of ileal pouches are used? 578
Anorectal Disease 578
14. What are anal fissures? 578
15. What disorders should be considered in patients with laterally situated anal fissures? 578
16. How are acute fissures managed? 578
17. What are the signs of a chronic anal fissure? What do they imply? 579
18. Which surgical procedures are available for treatment of a chronic anal fissure? 579
19. How are hemorrhoids classified? 579
20. How are acute hemorrhoids treated? 579
21. List several minimally invasive outpatient treatments of internal hemorrhoids. 579
22. Who is the patron saint of hemorrhoid sufferers? 579
23. How is an acute thrombosed external hemorrhoid best treated? 579
24. Explain the cause of anorectal abscesses andfistulas. 579
25. List the various types and locations of anorectalabscesses. 579
26. What is the best treatment for an anorectalabscess? 579
27. What is the Goodsall rule? 579
28. What is a seton? 580
29. What are the common indications for inserting a seton? 580
30. List new developments for treatment of anorectal fistulas. 580
31. When is anorectal suppurative disease especially dangerous? 580
32. What is Fournier gangrene? 580
33. Describe perianal Paget disease. 580
34. Which patient characteristics are associated with rectal prolapse? 580
35. What surgical options are available for rectal prolapse? 580
36. How is rectal prolapse handled in pediatric patients? 580
Colorectal Malignancies 580
37. What is the best way to stage rectal cancer? 580
38. When is endoscopic mucosal resection (EMR) indicated? 581
39. What are the indications for neoadjuvant (before surgery) and adjuvant (after surgery)therapy? 581
40. What is an abdominal perineal resection (APR), and when is it indicated? 581
Colon Cancer 581
41. What are the fundamental principals of colon resection for cancer? 581
42. Does laparoscopic surgery compromise the chance for a cure? 581
43. What are the pros and cons of laparoscopic versus open colectomy? 581
44. What are the findings of sigmoid volvulus on plain abdominal film and contrastenema? 581
45. How is a nonstrangulated sigmoid volvulus treated? 581
46. Why should elective surgery be performed after a successful endoscopic detorsionand decompression of a sigmoid volvulus? 582
47. Do colon perforations from colonoscopy mandate surgical repair? 582
48. What is Ogilvie syndrome? 582
49. What does plain radiographic study of the abdomen reveal in large bowel obstruction? 582
50. What radiologic findings are associated with gallstone ileus? 582
51. What does endometriosis have to do with the alimentary system? 582
52. What is a primary bowel obstruction? 582
53. How is postoperative ileus differentiated from postoperative SBO? 582
54. Is treatment of postoperative SBO different from treatment of SBO remote from surgery? 582
55. What is the most common cause of SBO? 582
56. Can adhesions be prevented? 582
57. What are the pathologic findings of late radiation enteritis? 582
58. What are general principles of managing radiation enteritis? 583
59. What treatments are available for bleeding radiation proctitis? 583
Websites 583
Bibliography 583
Chapter 79: Bariatric Surgery 584
1. What is the definition of obesity? 584
2. How is body fat relative to weight usually measured? 584
3. Describe the BMI classification system 584
4. What are the limitations of BMI? 584
5. What proportion of the U.S. adult population is considered overweight? 584
6. What proportion of the U.S. adult population is considered obese? 584
7. Are there health implications associated with a BMI of 30 kg/m2? 584
8. Can obesity lead to premature death? 584
9. How successful is nonsurgical treatment of obesity? 584
10. How is obesity best treated? 584
11. What was the NIH Consensus Statement? 585
12. List the contraindications to bariatric surgery 585
13. Categorize the surgical options for weight reduction 585
14. List the options for restrictive surgery 585
15. Describe the combined restrictive/malabsorptive option 585
16. What is the option for malabsorptive surgery? 586
17. The other category includes which procedures? 586
18. What are the weight loss expectations after each procedure? 586
19. Are these just cosmetic operations? 586
20. Does surgical weight loss translate to improved long-term survival? 586
21. Which comorbidity can have the most dramatic improvement? 586
22. How does the gastric bypass and biliopancreatic diversion cure diabetes prior to weight loss? 586
23. Can these changes in the gut hormonal milieu have a detrimental effect? 586
24. What are other complications after a gastric bypass? 586
25. How are anastomotic leaks handled? 586
26. How is an anastomotic stenosis treated? 587
27. What is a marginal ulcer, and how is it treated? 587
28. What are the vitamin/mineral deficiencies and potential long-term risks? 587
29. What is the most common surgical weight loss procedure in Europe and Australia? 587
30. Why would someone choose a lap band over a Roux-en-Y gastric bypass? 587
31. What are the specific complications after a lap band procedure? 587
32. How does the biliopancreatic diversion work? 587
33. Is there malnutrition, and how problematic is it? 587
34. Are there other health risks associated with biliopancreatic diversion? 587
35. Why would one choose biliopancreatic diversion? 587
36. What does preoperative surgical counseling entail with any procedure? 587
Bibliography 588
Chapter 80: Laparoscopic Surgery 589
1. When did laparoscopic surgery become a credible surgical option? 589
2. What are the advantages of laparoscopic surgery compared with open procedures? 589
3. What are the contraindications to laparoscopic surgery? 589
4. Does laparoscopic surgery preserve immune function? 589
5. What are the respiratory effects of pneumoperitoneum (planned intra-abdominal hypertension)? 589
6. What are the hemodynamic effects? 589
7. At 24 hours after open upper abdominal surgery using subcostal incisions, patients show a decrease in pulmonary function tests of nearly 50%. What decreases should be expected at 24 hours after laparoscopic cholecystectomy? 590
8. Should we routinely use prophylactic antibiotics for laparoscopic cholecystectomy? 590
9. What is the difference between the hepatocystic triangle and the triangle of Calot? 590
10. Which alternative gases can be used for laparoscopy? 590
11. List the advantages and disadvantages of using carbon dioxide (CO2) as an insufflation gas instead of other gases 591
12. A 9-year-old girl presents with a 2-month history of right upper quadrant abdominal pain that most commonly occurs after eating fatty foods and usually resolves in 30 minutes. She is afebrile, and the physical exam is unremarkable. Laboratory values, in 591
13. TThe HIDA scan demonstrated rapid filling of the gallbladder and unobstructed flow into the duodenum. Cholecystokinin (CCK) is administered, and the gallbladder ejection fraction (EF) is calculated at 30%. What is the most likely diagnosis? How should t 591
14. Summarize the key strategies for safe laparoscopic cholecystectomy. 591
15. Compare the rate of conversion from laparoscopic cholecystectomy to open cholecystectomy in patients with acute versus chronic cholecystitis 591
16. What pathophysiologic features of acute cholecystitis increase the likelihood of technical difficulties? 592
17. How does laparoscopic gastric bypass compare to open gastric bypass? 592
18. Is there any clearly defined benefit to laparoscopic appendectomy? 592
19. Is gangrenous or perforated appendicitis a contraindication to laparoscopic appendectomy? 592
20. Is laparoscopic antireflux surgery (LARS) justified for chronic gastroesophageal reflux disease (GERD)? 592
21. A 46-year-old woman with a BMI of 44 kg/m2 is referred for a laparoscopic antireflux procedure. She has been taking proton pump inhibitors for 20 years. Her symptoms are well controlled with medication; however, she prefers a surgical procedure if it wi 592
22. What are the benefits and drawbacks of laparoscopic versus open inguinal hernia repair? 592
23. What is the role of laparoscopic surgery for curable colon cancer? 592
24. A thin, 68-year-old woman with chronic obstructive pulmonary disease from 52 years of smoking undergoes laparoscopic cholecystectomy for acute cholecystitis. Because she has had a previous lower midline abdominal incision, you choose the open Hasson tec 593
25. Can laparoscopic cholecystectomy be done safely in the pregnant patient? 593
26. Can laparoscopic appendectomy be performed safely during pregnancy? 593
27. What are some technical considerations when performing laparoscopy on the pregnant patient? 593
28. What percentage of patients have free intra-abdominal air on upright radiograph 24 hours after laparoscopic procedure? 593
29. What are the indications and contraindications for laparoscopic adrenalectomy? What are the advantages of laparoscopic ... 594
30. How are bile spills managed? 594
Websites 594
Bibliography 594
Index 595